PDA

نسخه کامل مشاهده نسخه کامل : اتاق ریاضیات(طرح سؤالات)



صفحه ها : 1 2 3 [4] 5 6 7 8 9 10 11 12 13 14 15 16 17 18 19 20

mir@
25-04-2007, 22:57
سلام ببخشید سوالمو اینجا مطرح میکنم
استاد مدار منطقی یکی از دوستام این سوال رو مطرح کرده و گفته خودش دوارن جوونیاش 6 ماه روش فکر کرده تا جوابشو پیدا کرده حالا من میخوام از شما دوشتان این سوالو بپرسم تا شما در صورت امکان پاسخ منو بدید
سوال: 12 گلوله داریم که هیچ فرق ظاهری ندارند با هم فقط یکی از گلوله ها وزنش با بقیه فرق میکنه نه میدونیم بیشتر هست نه کمتر ، میخواهیم با 3 بار وزن کردن اون گلوله را مشخص کنیم باید چه کار کرد؟؟؟؟؟؟؟؟؟؟؟
راهنمایی به 3 گروه 4 تایی تقسیم میکنیم

ممنون

سلام دوست عزیز

اولاً بهت توصیه می کنم خودت روش فکر کنی. بسیار بسیار جالبه

اما جواب:

1- اول 12 سکه را به 4 دسته 3 تایی تقسیم میکنیم

2- در یک ترازو دو دسته را با هم وزن میکنیم اگه تعادل برقرا بود که یعنی سکه معیوب در این دسته نیست اما اگه تعادل برقرار نبود..

3- فرض کنیم تعادل برقرار نباشه نتیجه میگیرم که سکه معیوب جز یکی از این 6 سکه هستش مثلا فرض کنیم که کفه سمت راست سبکتر از کفه سمت چپ بود... (فرض کردیم!!)

4- خوب حالا اون یک دسته سکه که در مرحله 2 مطئن شدیم که سکه بین اونها نیست را در یک طرف و در طرف دیگه یکی از دو دسته سکه هایی که در مرحله 3 تست کردیم قرار میدهیم!! (فرض کنیم همون طرفی که سبکتر بود!!)

الف- اگه تعادل برقرار بود نتیجه میگیرم که سکه معیوب جز اون یکی دسته هستش که در مرحله 3 متوجه شدیم سنگینتره (نتیجه :سکه معیوب سنگینتر هستش!!)
ب- اگه تعادل برقرار نبود نتیجه میگیریم که سکه معیوب در همین دسته هستش و سبکتر میباشد..

5- خوب مساله حل شد الان ما فقط 3 سکه داریم و مطمئن هستیم که سکه معیوب سنگینتره و یا سبکتر (با توجه به مرحله 4)
کافیه 2 عدد از این سکه ها را باهم وزن کنیم اگه تعادل برقرار بود سکه معیوب اون یکی دیگه هستش!!
اگه تعادل برقرار نبود با توجه به مرحله 4 و مشخص شدن وضعیت سکه میتوان سکه معیوب را پیدا کرد...

Babak Majidzadeh
25-04-2007, 23:05
سلام

لطفا براي ديدن سوال به لينك زير برويد:

[ برای مشاهده لینک ، لطفا با نام کاربری خود وارد شوید یا ثبت نام کنید ]

اميدوارم درست عمل كرده باشم

mir@
25-04-2007, 23:07
از آقای مفیدی ممنونم که سوالم رو بستن ممنون!
اما دوباره میگم:
حد زیر را بدون استفاده از هم ارزی و هوپتال حل کنید:
Lim (x-sinx)/x^3 x-> 0

من یه چیزی می تئنم بگم عزیز جان:

از لحاظ بسط تیلور، سینوس حول نقطه صفر به صورت زیر بسط داده میشه:


[ برای مشاهده لینک ، لطفا با نام کاربری خود وارد شوید یا ثبت نام کنید ]

mofidy1
25-04-2007, 23:55
حد زیر را بدون استفاده از هم ارزی و هوپتال حل کنید:
Lim (x-sinx)/x^3 x-> 0

با سلام

روشی که دوستمان در پست 750 ([ برای مشاهده لینک ، لطفا با نام کاربری خود وارد شوید یا ثبت نام کنید ]) مطرح کردند، به روش هم ارزی تعمیم یافته معروف است. روش دیگری نیز وجود دارد به نام روش بازگشتی. قرار دهید:


[ برای مشاهده لینک ، لطفا با نام کاربری خود وارد شوید یا ثبت نام کنید ]

(البته به شرطی که حد موجود باشد.) با فرض x=3t می توان نوشت:


[ برای مشاهده لینک ، لطفا با نام کاربری خود وارد شوید یا ثبت نام کنید ]

نکته:

این مساله به خوبی نشان می دهد که نمی توان از این قانون مثلاً هم ارزی (که از آن مثل نقل و نبات در دبیرستان و پیش دانشگاهی استفاده می شود و حتی گاهی آنرا یکی از روشهای مدرن تست زنی می نامند) استفاده کرد. باید توجه کرد که همواره نمی توان وقتی x به سمت صفر میل کرد، به جای (sin(x همان x را قرار داد و جواب حد را به دست آورد (دقت کنید!!).

موفق باشید.

5 اردیبهشت 1386

Babak Majidzadeh
26-04-2007, 00:58
سلام

بدون بسط ثابت كنيد

[ برای مشاهده لینک ، لطفا با نام کاربری خود وارد شوید یا ثبت نام کنید ] ([ برای مشاهده لینک ، لطفا با نام کاربری خود وارد شوید یا ثبت نام کنید ])

mir@
26-04-2007, 13:43
سلام جناب مفیدی،

این روش بازگشتی واقعاً عالی بود. خیلی خیلی استفاده کردم.
این ریاضی هم عجب چیزیه. هر مسئله 1000 راه حل داره یکی از یکی جالب تر.

درود بر شما :11:

leon1370
26-04-2007, 22:42
با سلام

آقای مفیدی اگر می شود فرمول های زیر را برام اثبات و بنویسین؟ ( با تشکر )

s جانبی هرم:
s کل :

آیا فرمولی برای اعداد اول وجود دارد؟ من در اخبار شنیدم که یک ایرانی آن را اثبات کرده؟

با تشکر از زحمات شما

mir@
26-04-2007, 23:41
با سلام

آقای مفیدی اگر می شود فرمول های زیر را برام اثبات و بنویسین؟ ( با تشکر )

s جانبی هرم:
s کل :

آیا فرمولی برای اعداد اول وجود دارد؟ من در اخبار شنیدم که یک ایرانی آن را اثبات کرده؟

با تشکر از زحمات شما

سلام

می دانیم که اگر منحنی y تابعی از x حول محور x دوران کند و a<x<b سطح جانبی رویه حاصل از فرمول زیر به دست می آید:


[ برای مشاهده لینک ، لطفا با نام کاربری خود وارد شوید یا ثبت نام کنید ]

حالا شکل زیر را در نظر بگیرید


[ برای مشاهده لینک ، لطفا با نام کاربری خود وارد شوید یا ثبت نام کنید ]

اگر خط قرمز حول محور x دوران کند مخروطی با ارتفاع h و شعاع قاعده r حاصل می شود. معادله خط عبارتست از:


[ برای مشاهده لینک ، لطفا با نام کاربری خود وارد شوید یا ثبت نام کنید ]

اگر این معادله را در فرمول اول قرار دهیم داریم:


[ برای مشاهده لینک ، لطفا با نام کاربری خود وارد شوید یا ثبت نام کنید ]

یعنی سطح جانبی مخروط مساوی است با نصف محیط قاعده ضرب در اندازه مولد

سطح کل هم که مساوی است با سطح جانبی به اضافه مساحت دایره قاعده

در مورد اعداد اول هم من اطلاعی ندارم. ولی بعیده!

ببخشید که فضولی کردم. امیدوارم اگر اشکالی داره استاد مفیدی اصلاح کنند.

ارادتمند :11:

پاکر
27-04-2007, 01:01
نه مثل اینکه با یه آریایی طرفم!
(راه حلش رو بلد بودمmir@ !)

پاکر
27-04-2007, 01:20
آقای مفیدی ممنون میشم اگه منو کمی راهنمایی کنین (فقط کمی!)
چه طور میشه با روش های کتاب پیش دانشگاهی تابع اولیه تابع زیر رو پیدا کرد؟
S tan(x)^1/2
(بدیهیه که کل تابع زیر رادیکال)

new_boys333
27-04-2007, 09:59
سلام دوست عزیز

اولاً بهت توصیه می کنم خودت روش فکر کنی. بسیار بسیار جالبه

اما جواب:

1- اول 12 سکه را به 4 دسته 3 تایی تقسیم میکنیم

2- در یک ترازو دو دسته را با هم وزن میکنیم اگه تعادل برقرا بود که یعنی سکه معیوب در این دسته نیست اما اگه تعادل برقرار نبود..

3- فرض کنیم تعادل برقرار نباشه نتیجه میگیرم که سکه معیوب جز یکی از این 6 سکه هستش مثلا فرض کنیم که کفه سمت راست سبکتر از کفه سمت چپ بود... (فرض کردیم!!)

4- خوب حالا اون یک دسته سکه که در مرحله 2 مطئن شدیم که سکه بین اونها نیست را در یک طرف و در طرف دیگه یکی از دو دسته سکه هایی که در مرحله 3 تست کردیم قرار میدهیم!! (فرض کنیم همون طرفی که سبکتر بود!!)

الف- اگه تعادل برقرار بود نتیجه میگیرم که سکه معیوب جز اون یکی دسته هستش که در مرحله 3 متوجه شدیم سنگینتره (نتیجه :سکه معیوب سنگینتر هستش!!)
ب- اگه تعادل برقرار نبود نتیجه میگیریم که سکه معیوب در همین دسته هستش و سبکتر میباشد..

5- خوب مساله حل شد الان ما فقط 3 سکه داریم و مطمئن هستیم که سکه معیوب سنگینتره و یا سبکتر (با توجه به مرحله 4)
کافیه 2 عدد از این سکه ها را باهم وزن کنیم اگه تعادل برقرار بود سکه معیوب اون یکی دیگه هستش!!
اگه تعادل برقرار نبود با توجه به مرحله 4 و مشخص شدن وضعیت سکه میتوان سکه معیوب را پیدا کرد...


ممنون دوست خوبم فقط من یکم تو مرحله 4 گیر کردم گلوله های مرحله 2 که مطمئن شدیم گلوله های ما بین اونا نیست یعنی اونایی یکی از اون دو دسته ای که وزن نکردیم؟؟؟؟؟؟؟؟؟؟؟؟؟؟؟؟؟؟؟ ؟؟؟؟؟؟؟؟؟؟؟؟؟؟؟؟؟؟؟؟؟؟؟؟؟ ؟؟ چون تو مر حله 2 وقتی ما دو دسته رو وزن کردیم و تعادل بر قرار نبود( فرض)
بعد یک سوال دیگه اگه مرحله اول اگه تعادل بر قرار بود باز با همین 3 بار وزن کردن ( که یکیش هم استفاده شده) میشه تا ته مسئله رو رفت و جواب رو گرفت؟؟؟؟؟؟؟؟؟؟؟؟


یک دنیا ممنون خیلی مخلصم

new_boys333
27-04-2007, 10:12
سلام دوست عزیز

اولاً بهت توصیه می کنم خودت روش فکر کنی. بسیار بسیار جالبه

اما جواب:

1- اول 12 سکه را به 4 دسته 3 تایی تقسیم میکنیم

2- در یک ترازو دو دسته را با هم وزن میکنیم اگه تعادل برقرا بود که یعنی سکه معیوب در این دسته نیست اما اگه تعادل برقرار نبود..

3- فرض کنیم تعادل برقرار نباشه نتیجه میگیرم که سکه معیوب جز یکی از این 6 سکه هستش مثلا فرض کنیم که کفه سمت راست سبکتر از کفه سمت چپ بود... (فرض کردیم!!)

4- خوب حالا اون یک دسته سکه که در مرحله 2 مطئن شدیم که سکه بین اونها نیست را در یک طرف و در طرف دیگه یکی از دو دسته سکه هایی که در مرحله 3 تست کردیم قرار میدهیم!! (فرض کنیم همون طرفی که سبکتر بود!!)

الف- اگه تعادل برقرار بود نتیجه میگیرم که سکه معیوب جز اون یکی دسته هستش که در مرحله 3 متوجه شدیم سنگینتره (نتیجه :سکه معیوب سنگینتر هستش!!)
ب- اگه تعادل برقرار نبود نتیجه میگیریم که سکه معیوب در همین دسته هستش و سبکتر میباشد..

5- خوب مساله حل شد الان ما فقط 3 سکه داریم و مطمئن هستیم که سکه معیوب سنگینتره و یا سبکتر (با توجه به مرحله 4)
کافیه 2 عدد از این سکه ها را باهم وزن کنیم اگه تعادل برقرار بود سکه معیوب اون یکی دیگه هستش!!
اگه تعادل برقرار نبود با توجه به مرحله 4 و مشخص شدن وضعیت سکه میتوان سکه معیوب را پیدا کرد...


من فکر کردم دیدم اگه مرحله یک که دو دسته 3 تایی رو وزن کردیم و اگه تعادل بر قرار باشه باید 2 دسته دیگه رو امتحان کنیم و بعد فرض میگیریم سبکه رو انتخاب کنیم و مانند روش شما پیش بریم 4 بار وزن کردن میشه در صورتی که ما فقط مجاز به 3 بار وزن کردن هستیم باید چه کرد؟؟؟؟؟؟؟؟؟؟؟؟؟؟؟؟؟؟؟؟؟؟ ؟؟؟؟؟؟؟؟؟؟؟؟؟؟؟؟؟؟؟؟؟؟؟؟؟ ؟؟؟؟؟؟؟؟؟؟؟؟؟؟؟
ممنون

Jalal
27-04-2007, 10:33
از بچه هاي رياضي دان كسي هست كار مارو راه بندازه ([ برای مشاهده لینک ، لطفا با نام کاربری خود وارد شوید یا ثبت نام کنید ])

leon1370
27-04-2007, 13:08
با سلام مجدد

آقای mir@ من سطح جانبی هرم را خواستم نه مخروط !
لطفا اگر بلدید برام بنویسین؟
آقای مفیدی اگر می شود راهنمایی کنید؟

mir@
27-04-2007, 15:03
ممنون دوست خوبم فقط من یکم تو مرحله 4 گیر کردم گلوله های مرحله 2 که مطمئن شدیم گلوله های ما بین اونا نیست یعنی اونایی یکی از اون دو دسته ای که وزن نکردیم؟؟؟؟؟؟؟؟؟؟؟؟؟؟؟؟؟؟؟ ؟؟؟؟؟؟؟؟؟؟؟؟؟؟؟؟؟؟؟؟؟؟؟؟؟ ؟؟ چون تو مر حله 2 وقتی ما دو دسته رو وزن کردیم و تعادل بر قرار نبود( فرض)
بعد یک سوال دیگه اگه مرحله اول اگه تعادل بر قرار بود باز با همین 3 بار وزن کردن ( که یکیش هم استفاده شده) میشه تا ته مسئله رو رفت و جواب رو گرفت؟؟؟؟؟؟؟؟؟؟؟؟


یک دنیا ممنون خیلی مخلصم


من فکر کردم دیدم اگه مرحله یک که دو دسته 3 تایی رو وزن کردیم و اگه تعادل بر قرار باشه باید 2 دسته دیگه رو امتحان کنیم و بعد فرض میگیریم سبکه رو انتخاب کنیم و مانند روش شما پیش بریم 4 بار وزن کردن میشه در صورتی که ما فقط مجاز به 3 بار وزن کردن هستیم باید چه کرد؟؟؟؟؟؟؟؟؟؟؟؟؟؟؟؟؟؟؟؟؟؟ ؟؟؟؟؟؟؟؟؟؟؟؟؟؟؟؟؟؟؟؟؟؟؟؟؟ ؟؟؟؟؟؟؟؟؟؟؟؟؟؟؟
ممنون

سلام دوست عزیز،
اون جواب قبلی کاملاً غلط بود. به همون دلیلی که خودت گفتی. اگر در وزن کردن اول هر دو مساوی در بیان دیگه کاری نمیشه کرد. اما جواب درست.

سکه ها را از 1 تا 12 نام گذاری می کنیم.

سه بار وزن می کنیم به صورت زیر:

--------------سمت چپ ----------سمت راست
توزین اول: 1-2-3-10 ------------ 4-5-6-11
توزین دوم:-1-2-3-11--------------- 7-8-9-10
توزین سوم: 1-4-7-10--------------3-6-9-12

حالا حالتی رو که طرف راست سنگین تر بشه R می نامیم، طرف چپ رو L و در صورت تعادل B حالات زیر ممکن است و سکه معیوب به صورت زیر مشخص می شود.


L L L 1 heavier
L L R 3 heavier
L L B 2 heavier
L R L 10 heavier
L R B 11 lighter
L B L 6 lighter
L B R 4 lighter
L B B 5 lighter
R L R 10 lighter
R L B 11 heavier
R R L 3 lighter
R R R 1 lighter
R R B 2 lighter
R B L 4 heavier
R B R 6 heavier
R B B 5 heavier
B L L 9 lighter
B L R 7 lighter
B L B 8 lighter
B R L 7 heavier
B R R 9 heavier
B R B 8 heavier
B B L 12 lighter
B B R 12 heavier


توجه شود که سه حالت LRRو RLL و BBB امکان پذیر نیست.


همون طور که می بینید، این اصلاً یک مسئله پیش پا افتاده نیست و بسیار روش بحث شده و حل کردنش هم کار من و شما نیست. فکر کنم یه چیزی در حد رساله ریاضی دکترا باشه:27: .

:11:

mir@
27-04-2007, 15:34
یک معمای جالب و دشوار برای دوست عزیزی که سوال 12 گلوله رو مطرح کردند:

5 نفر دزد دریایی 1000 سکه طلا رو از ته آب می کشن بیرون. حالا می خوان اینا رو بین خودشون تقسیم کنن. رتبه و درجه این افراد هم از 1 تا 5 هست. یعنی نفر 1 از همه رتبه اش بالاتره تا نفر پنجم.

به این ترتیب عمل می کنند که از کسی که رتبه اش از همه پایین تره می پرسن تو میگی چه جوری تقسیم کنیم؟ اگر پیشنهاد قبول بشه که عمل میشه و کار تمومه. اگر رد بشه، اون شخص به دهان کوسه انداخته میشه :18: و نوبت به نفر چهارم می رسه و الی آخر.

پیشنهادی پذیرفته میشه که اکثریت (یعنی نصف به اضافه یک) قبولش کنن.

دزدان دریایی هر 5 تاشون این 3 ویژگی اخلاقی رو در بالاترین مرتبه دارند:


بسیار باهوش

بسیار خونخوار

بسیار حریص

حالا شما اگر جای نفر پنجم بودید چه راه حلی رو پیشنهاد می دادید برای تقسیم تا همه قبول کنند و شما نمیرید؟

:11:

mofidy1
27-04-2007, 18:56
با سلام

فرض کنید G گروهی کامل باشد( یعنی مشتق گروه برابر خود گروه باشد). ثابت کنید هر زیرگروه نرمال دوری G زیر مجموعه ای از مرکز G است.

موفق باشید.

ارسال متن: دوشنبه 3 اردیبهشت 1386

با سلام

فرض کنید H زیر گروه نرمال دوری G باشد. می دانیم که زیر گروهی مانند K از اتومورفیسم های H چنان موجود است که


[ برای مشاهده لینک ، لطفا با نام کاربری خود وارد شوید یا ثبت نام کنید ]

چون H در G نرمال است، لذا


[ برای مشاهده لینک ، لطفا با نام کاربری خود وارد شوید یا ثبت نام کنید ]

بنابرفرض H دوری است، لذا گروه اتومورفیسم های H آبلی و در نتیجه K نیز آبلی است. بنابر این


[ برای مشاهده لینک ، لطفا با نام کاربری خود وارد شوید یا ثبت نام کنید ]

که نتیجه می دهد H زیرگروهی از مرکز G است.

موفق باشید.

ارسال متن: جمعه 7 اردیبهشت 1386

mir@
27-04-2007, 20:41
با سلام مجدد

آقای mir@ من سطح جانبی هرم را خواستم نه مخروط !
لطفا اگر بلدید برام بنویسین؟
آقای مفیدی اگر می شود راهنمایی کنید؟

اشتباه از من بود.

سطح جانبی هرم برابر است با مجموع سطوح جانبی آن :27:

فکر می کنم بدیهی باشه. این دقیقاً فرمولیه که تو کتاب ها نوشته. :11:

Babak Majidzadeh
27-04-2007, 23:30
با سلام

روشی که دوستمان در پست 750 ([ برای مشاهده لینک ، لطفا با نام کاربری خود وارد شوید یا ثبت نام کنید ]) مطرح کردند، به روش هم ارزی تعمیم یافته معروف است. روش دیگری نیز وجود دارد به نام روش بازگشتی. قرار دهید:


[ برای مشاهده لینک ، لطفا با نام کاربری خود وارد شوید یا ثبت نام کنید ]

(البته به شرطی که حد موجود باشد.) با فرض x=3t می توان نوشت:


[ برای مشاهده لینک ، لطفا با نام کاربری خود وارد شوید یا ثبت نام کنید ]

نکته:

این مساله به خوبی نشان می دهد که نمی توان از این قانون مثلاً هم ارزی (که از آن مثل نقل و نبات در دبیرستان و پیش دانشگاهی استفاده می شود و حتی گاهی آنرا یکی از روشهای مدرن تست زنی می نامند) استفاده کرد. باید توجه کرد که همواره نمی توان وقتی x به سمت صفر میل کرد، به جای (sin(x همان x را قرار داد و جواب حد را به دست آورد (دقت کنید!!).

موفق باشید.

5 اردیبهشت 1386

سلام

البته اين روش به طور ضمني فرض مي كند كه حد مورد نظر وجود دارد.

shilaa
28-04-2007, 08:15
سوال منو جواب ندادین

پرسیدم کسی هست روش حل معادله چندجمله ای رو به روش نیوتن و استفاده از قاعده هورنر بلد باشه؟؟؟؟

new_boys333
28-04-2007, 12:46
همون طور که می بینید، این اصلاً یک مسئله پیش پا افتاده نیست و بسیار روش بحث شده و حل کردنش هم کار من و شما نیست. فکر کنم یه چیزی در حد رساله ریاضی دکترا باشه:27: .

:11:

سلام و ممنون دوست خوبم
یعنی نمیشه حلش کرد؟؟؟؟؟؟؟؟؟؟؟؟؟؟؟؟؟؟؟؟؟
سایت خارجی که مربوط به ریاضی باشه یا هوش کسی نمی شناسه؟؟؟؟؟؟؟؟؟؟؟؟؟؟؟؟؟

mofidy1
28-04-2007, 13:09
با سلام

خط d و نقاط A و B را که در یک طرف این خط قرار دارند، در نظر بگیرید. نقطه P روی این خط را به گونه ای بیابید که PA+PB کوتاهترین مقدار ممکن باشد. (ادعای خود را ثابت کنید.)

موفق باشید.

ارسال متن: شنبه 8 اردیبهشت 1386

mir@
28-04-2007, 14:48
سلام و ممنون دوست خوبم
یعنی نمیشه حلش کرد؟؟؟؟؟؟؟؟؟؟؟؟؟؟؟؟؟؟؟؟؟
سایت خارجی که مربوط به ریاضی باشه یا هوش کسی نمی شناسه؟؟؟؟؟؟؟؟؟؟؟؟؟؟؟؟؟

سلام

عزیزم حلش رو که بالاتر نوشتم. با رنگ آبی و ضخیم.

دقت کن دیگه.

در ضمن میتونی برای یافتن منابع بیشتر راجع به این سوال عبارت "Weighing 12 coins" رو از طریق google سرچ کنی. :11:

mir@
28-04-2007, 15:07
با سلام

خط d و نقاط A و B را که در یک طرف این خط قرار دارند، در نظر بگیرید. نقطه P روی این خط را به گونه ای بیابید که PA+PB کوتاهترین مقدار ممکن باشد.

موفق باشید.

ارسال متن: شنبه 8 اردیبهشت 1386

سلام

تصویر نقطه B را در آن طرف خط پیدا می کنیم. بعد خطی از A به B' وصل می کنیم. نقطه برخورد خط A'B با خط اصلی نقطه P خواهد بود.

اثبات هم لازمه؟

:11:

پاکر
29-04-2007, 00:22
آقای مفیدی گرفتی مارو چرا جواب سوالم رو نمیدی
حالا چون ما از اون ستاره ها نداریم جواب نمیدید!

پاکر
29-04-2007, 00:51
سوالم سرجاش هست
(آقای مفیدی ممنون میشم اگه منو کمی راهنمایی کنین (فقط کمی!)
چه طور میشه با روش های کتاب پیش دانشگاهی تابع اولیه تابع زیر رو پیدا کرد؟
S tan(x)^1/2 dx
(بدیهیه که کل تابع زیر رادیکال) )

اما مرد میخوام اینو حل کنه

a^3 +b^3)^1/3 = c)

and

c=a,b,0 نباشه و a و b متعلق به z

ثابت کنید cبه مجموعه اعداد صحیح متعلق نیست.

new_boys333
29-04-2007, 21:04
سلام

عزیزم حلش رو که بالاتر نوشتم. با رنگ آبی و ضخیم.

دقت کن دیگه.

در ضمن میتونی برای یافتن منابع بیشتر راجع به این سوال عبارت "Weighing 12 coins" رو از طریق google سرچ کنی. :11:

ممنون دوست بسیار خوبم فقط میشه بگی این جوابو از کجا آوردی و چرا به این گونه وزن کردیم سکه ها رو ؟ شماره سکه ها رو میگم؟
ممنون

Iron
30-04-2007, 02:38
سلام ببخشید سوالمو اینجا مطرح میکنم
استاد مدار منطقی یکی از دوستام این سوال رو مطرح کرده و گفته خودش دوارن جوونیاش 6 ماه روش فکر کرده تا جوابشو پیدا کرده حالا من میخوام از شما دوشتان این سوالو بپرسم تا شما در صورت امکان پاسخ منو بدید
سوال: 12 گلوله داریم که هیچ فرق ظاهری ندارند با هم فقط یکی از گلوله ها وزنش با بقیه فرق میکنه نه میدونیم بیشتر هست نه کمتر ، میخواهیم با 3 بار وزن کردن اون گلوله را مشخص کنیم باید چه کار کرد؟؟؟؟؟؟؟؟؟؟؟
راهنمایی به 3 گروه 4 تایی تقسیم میکنیم

ممنون
البته میشه مساله رو جور دیگه ای هم حل کرد که با راهنمایی داده شده همخوانی داره. سکه ها رو به سه گروه چهارتایی تقسیم می کنیم.
1- گروه اول و دوم رو در دو کفه قرار می دیم.
اگه گروه اول و دوم هموزن بودند:
معلوم میشه گلوله غیر هموزن در گروه گروه سومه. دو گلوله از گروه اول رو در کفه چپ و دو گلوله از گروه سوم رو در کفه راست می گزاریم. اگر هموزن نبودند یکی از دو گلوله گروه سوم که وزن شده گلوله مورد نظر است. در غیر اینصورت گلوله مورد نظر یکی از دو گلوله وزن نشده گروه سوم است. حال باید با یک بار توزین گلوله مورد نظر را از بین دو گلوله بیابیم. برای این منظور یکی از گلوله ها را با یکی از گلوله های گروه اول وزن می کنیم. اگر هموزن نبودند پس گلوله مورد نظر همین گلوله است و درغیر اینصورت گلوله دیگر است.


حال اگر در مرحله 1 دو گروه هموزن نبودند:
فرض می کنیم کفه سمت راست سنگینتر باشد. از کفه سمت راست دو گلوله را خارج می کنیم. همچنین یکی از گلوله های کفه راست را در کفه چپ و دوتا از گلوله های کفه چپ را در کفه راست می گذاریم. یکی از حالتهای الف،ب یا ج رخ خواهد داد

الف-اگر کفه ها به تعادل برسند، گلوله غیر هموزن در بین دو گلوله خارج شده قرار دارد و قطعا این گلوله سنگینتر از سایر گلوله هاست.(چون هر دو از کفه سمت راست که سنگینتر بوده برداشته شده اند) با مقایسه وزن این دو گلوله، گلوله سنگینتر را پیدا می کنیم.

ب-اگر کفه سمت چپ سنگینتر باشد، گلوله مورد نظر در بین سه گلوله جابجا شده قرار دارد. (چون کفه سنگینتر جابجا شده است) از آنجاکه دو گلوله از گلوله های جابجا شده در کفه سمت راستند (کفه سبکتر)، پس اگر گلوله غیر هموزن یکی از این دو گلوله باشد باید سبکتر از سایر گلوله ها باشد. وزن ایندو را با هم مقایسه می کنیم و هرکدام که سبکتر باشد گلوله مورد نظر است. اگر هموزن باشند گلوله سوم گلوله مورد نظر است.

ج- اگر همچنان کفه سمت راست سنگینتر باشد، پس گلوله مورد نظر بین سه گلوله جابجا نشده و خارج نشده است. از آنجاکه دو تا از این گلوله ها در کفه سمت چپند (کفه سبکتر)، وزن آنها را باهم مقایسه کرده و گلوله سبکتر را انتخاب می کنیم. اگر هموزن بودند گلوله سوم گلوله مورد نظر است.

Iron
30-04-2007, 02:41
یک معمای جالب و دشوار برای دوست عزیزی که سوال 12 گلوله رو مطرح کردند:

5 نفر دزد دریایی 1000 سکه طلا رو از ته آب می کشن بیرون. حالا می خوان اینا رو بین خودشون تقسیم کنن. رتبه و درجه این افراد هم از 1 تا 5 هست. یعنی نفر 1 از همه رتبه اش بالاتره تا نفر پنجم.

به این ترتیب عمل می کنند که از کسی که رتبه اش از همه پایین تره می پرسن تو میگی چه جوری تقسیم کنیم؟ اگر پیشنهاد قبول بشه که عمل میشه و کار تمومه. اگر رد بشه، اون شخص به دهان کوسه انداخته میشه :18: و نوبت به نفر چهارم می رسه و الی آخر.

پیشنهادی پذیرفته میشه که اکثریت (یعنی نصف به اضافه یک) قبولش کنن.

دزدان دریایی هر 5 تاشون این 3 ویژگی اخلاقی رو در بالاترین مرتبه دارند:


بسیار باهوش

بسیار خونخوار

بسیار حریص

حالا شما اگر جای نفر پنجم بودید چه راه حلی رو پیشنهاد می دادید برای تقسیم تا همه قبول کنند و شما نمیرید؟

:11:
ببخشید. اینو برای یکی دیگه طرح کرده بودید ولی من جواب می دم

یک سکه به نفر دوم. یک سکه به نفر سوم. 998 سکه به نفر چهارم می دم.
نفر دوم و سوم هیچگاه راضی نمیشن که تعداد نفرات به سه نفر برسه. چراکه در این صورت تنها با رای مخالف نفر اول هردوی آنها به دریا انداخته خواهند شد و نفر اول همه سکه ها رو تصاحب خواهد کرد. بنابراین در چنین حالتی یا باید همه سکه ها رو به نفر اول بدن یا بمیرن. از این رو اگر نفر پنجم کشته شود، نفر چهارم می تواند با دادن تنها یک سکه به نفرات دوم و سوم، رای موافق آنها را بگیرد. (چراکه اگر مخالفت کنند و نفر چهارم کشته شود همان یک سکه هم به آنها نمی رسد)
پس نفر چهارم حداکثر می تواند 998 سکه داشته باشد و نفرات دوم و سوم حداکثر یک سکه و لذا نفر پنجم با پیشنهاد چنین نحوه تقسیمی، می تواند رای مثبت این سه نفر را بدست آورد و زنده بماند.

Iron
30-04-2007, 03:42
البته شاید جاب پست قبلیم یجورایی مشکل داشته باشه. چون اگه چهار نفر به دریا انداخته بشن و فقط نفر اول بمونه، بازهم نمیتونه به اکثریت برسه. چون بنابر تعریف اکثریت برابر میشه با یک و نیم نفر در حالیکه فقط یه نفر باقی مونده و ناچار باید خودش بپره وسط کوسه ها. :31:
اگر فرض کنیم امکان رخ دادن چنین اتفاقی وجود داره، بنابراین اگر تنها دو نفر بمونن، نفر اول با هر پیشنهادی که نفر دوم بده موافقت می کنه و تمامی سکه ها به نفر دوم خواهد رسید.
اگرسه نفر بمونن هم نفر دوم با هر پیشنهادی که نفر سوم بده مخالفت می کنه و نفر سوم به دریا انداخته میشه مگر آنکه پیشنهاد بده تمامی سکه ها به نفر دوم برسه و به این ترتیب سر نفر اول و سوم بی کلاه خواهد ماند. پس این دو نفر بهیچوجه حاضر نیستند نفر چهارم به دریا انداخته بشه.
بنابراین اگر تنها چهار نفر باقی مونده باشن، نفر چهارم با پیشنهاد یک سکه به نفرات اول و سوم میتونه رای مثبت اونارو بدست بیاره و بقیه سکه ها رو برای خودش برداره.
همچنین نفر پنجم با ارائه همین پیشنهاد (نفر اول و سوم هر کدام یک سکه و نفر چهارم 998 سکه) میتونه زنده بمونه.

mir@
30-04-2007, 10:53
آفرین Iron عزیز،

معلومه قابلیت زیادی در سرچ داری مهندس.:31:

ارادتمند :11:

mofidy1
01-05-2007, 17:34
آقای مفیدی ممنون میشم اگه منو کمی راهنمایی کنین (فقط کمی!)
چه طور میشه با روش های کتاب پیش دانشگاهی تابع اولیه تابع زیر رو پیدا کرد؟
S tan(x)^1/2
(بدیهیه که کل تابع زیر رادیکال)


با سلام

ظاهرا منظور شما محاسبه ی انتگرال زیر است:


[ برای مشاهده لینک ، لطفا با نام کاربری خود وارد شوید یا ثبت نام کنید ]

با تغییر متغیر زیر شروع کنید


[ برای مشاهده لینک ، لطفا با نام کاربری خود وارد شوید یا ثبت نام کنید ]

و به این انتگرال برسید:


[ برای مشاهده لینک ، لطفا با نام کاربری خود وارد شوید یا ثبت نام کنید ]

برای حل آن نیز، اتحاد زیر را در نظر بگیرید:


[ برای مشاهده لینک ، لطفا با نام کاربری خود وارد شوید یا ثبت نام کنید ]

سپس از روش تجزیه کسر استفاده کنید.

(البته برای حل انتگرال دوم روش دیگری نیز وجود دارد.)

موفق باشید.

11 اردیبهشت 1386

پاکر
03-05-2007, 00:46
ممنون! و بازم ممنون!

Iron
03-05-2007, 22:37
با سلام

خط d و نقاط A و B را که در یک طرف این خط قرار دارند، در نظر بگیرید. نقطه P روی این خط را به گونه ای بیابید که PA+PB کوتاهترین مقدار ممکن باشد. (ادعای خود را ثابت کنید.)

موفق باشید.

ارسال متن: شنبه 8 اردیبهشت 1386

[ برای مشاهده لینک ، لطفا با نام کاربری خود وارد شوید یا ثبت نام کنید ]

پاره خط BC را بصورتی رسم می کنیم که که خط d عمود منصف آن باشد. پس همواره PC=PB.
بنابر این PC+AP=PB+AP.
بنابر قضیه حمار، کمترین مقدار AP+PC برابر است با طول پاره خط AC . بنابراین در حالتیکه P نقطه تقاطع پاره خط AC و خط d باشد، PC+AP=PB+AP کمترین مقدار خود را دارد. :31:

سلام بر ریاضیات
06-05-2007, 15:07
ثابت کنید اگر تابع F در نقطه صفر پیوسته باشد و f(x)2+f(y)2=f(x+y)+f(x-y)1
در تمام اعداد حقیقی پیوسته است.

سلام بر ریاضیات
06-05-2007, 15:09
ثابت کنید اگر تابع F در نقطه صفر پیوسته باشد و f(x)2+f(y)2=f(x+y)+f(x-y)1
در تمام اعداد حقیقی پیوسته است.

Hamid69
06-05-2007, 20:16
سلام.
چه طور میشه فهمید یه تابع مثلثاتی مثل y=sinx-1 مجانب داره یا نه؟(البته به جز راه حدی)

mofidy1
07-05-2007, 16:45
با سلام

خط d و نقاط A و B را که در یک طرف این خط قرار دارند، در نظر بگیرید. نقطه P روی این خط را به گونه ای بیابید که PA+PB کوتاهترین مقدار ممکن باشد. (ادعای خود را ثابت کنید.)

موفق باشید.

ارسال متن: شنبه 8 اردیبهشت 1386

با سلام

از Iron که در پست 782 ([ برای مشاهده لینک ، لطفا با نام کاربری خود وارد شوید یا ثبت نام کنید ]) مساله را حل کردند، تشکر می کنم. البته mir@ نیز در پست 772 جواب مساله را دادند، اما آنرا اثبات نکردند. راه حل زیر همان راه حل Iron است با کمی تغییر جملات:


[ برای مشاهده لینک ، لطفا با نام کاربری خود وارد شوید یا ثبت نام کنید ]

پاره خط BC را بصورتی رسم می کنیم که که خط d عمود منصف آن باشد. پس همواره PC=PB.
بنابر این PC+AP=PB+AP.
چون کوتاهترین فاصله بین دو نقطه، اندازه ی پاره خطی است که آن دو را به یکدیگر وصل می کند، بنابراین PC+AP=PB+AP کمترین مقدار خود را دارد.

موفق باشید.

ارسال متن: دوشنبه 17 اردیبهشت 1386

mofidy1
07-05-2007, 16:53
با سلام

ثابت کنید در میان اعداد روبرو، عدد اولی وجود ندارد: 10001, 100010001, 1000100010001,...

موفق باشید.

ارسال متن: دوشنبه 17 اردیبهشت 1386

leon1370
07-05-2007, 23:06
با سلام
کسی اگر می تونه لطفا فرمول زیر را برام اثبات کنه؟ با تشکر
n
c(n,0)+ c(n,1) + c(n,2( + ... +c(n,n) =2


لطفا کمک کنید

mir@
08-05-2007, 10:33
با سلام
کسی اگر می تونه لطفا فرمول زیر را برام اثبات کنه؟ با تشکر
n
c(n,0)+ c(n,1) + c(n,2( + ... +c(n,n) =2


لطفا کمک کنید

سلام برادر،
نمیدونم این اثبات هست یا نه ولی به هر حال

می دانیم که:


[ برای مشاهده لینک ، لطفا با نام کاربری خود وارد شوید یا ثبت نام کنید ]

حالا در فرمول بالا اگر قرار بدید a=1 و b=1 می بینید که ای دل غافل! شد همون چیزی که می خواستید.

mir@
08-05-2007, 10:51
با سلام

ثابت کنید در میان اعداد روبرو، عدد اولی وجود ندارد: 10001, 100010001, 1000100010001,...

موفق باشید.

ارسال متن: دوشنبه 17 اردیبهشت 1386


حل مسئله هفته پنجاهم

در معادله ab=cd
با فرض اینکه b اول باشد و مساوی با هیچ یک از c و d نباشد، بنابراین b باید عاملی از c یا d باشد و
a=(c/b)d یا a=(d/b)c
در هر صورت طرف راست هر دو معادله بالا بزرگتر از 1 است و لذا a نمی تواند اول باشد.


[ برای مشاهده لینک ، لطفا با نام کاربری خود وارد شوید یا ثبت نام کنید ]

در معادله بالا 101 اول است و وقتی i>1 هیچ یک از فاکتورهای سمت راست مساوی 101 نخواهد بود. بنابراین طبق بحث بالا، اگر i>1 باشد هیچ عددی به صورت [ برای مشاهده لینک ، لطفا با نام کاربری خود وارد شوید یا ثبت نام کنید ] نمی تواند اول باشد.

اگر i=1 معادله بالا تبدیل می گردد به 101*10001=101*10001 که خود چیزی را نشان نمی دهد. و ضمناً 10001=73*137

بدین ترتیب اثبات به پایان می رسد.:11:

mofidy1
12-05-2007, 18:01
با سلام

ثابت کنید در میان اعداد روبرو، عدد اولی وجود ندارد: 10001, 100010001, 1000100010001,...

موفق باشید.

ارسال متن: دوشنبه 17 اردیبهشت 1386

با سلام

با تشکر از mir@ که در پست 790 به حل مساله پرداختند.

اگر قرار دهید x=10 دنباله ی مطرح شده، به صورت زیر در می آید:


[ برای مشاهده لینک ، لطفا با نام کاربری خود وارد شوید یا ثبت نام کنید ]

اگر n عددی فرد باشد، مثلاً n=2m+1 ، خواهیم داشت:


[ برای مشاهده لینک ، لطفا با نام کاربری خود وارد شوید یا ثبت نام کنید ]

که نشان می دهد این عدد اول نیست .( توجه کنید که در این حالت m=0 یا m>0).

اگر n عددی زوج باشد، مثلاً n=2m، آنگاه:


[ برای مشاهده لینک ، لطفا با نام کاربری خود وارد شوید یا ثبت نام کنید ]

که باز نشان می دهد عدد بالا مرکب است.

موفق باشید.

ارسال متن: شنبه 22 اردیبهشت 1386

mofidy1
12-05-2007, 18:40
با سلام

مجموعه ی دلخواه 10 عضوی از اعداد طبیعی کوچکتر از 100 را در نظر بگیرید. ثابت کنید که می توان دو زیر مجموعه ی ناتهی مجزا ( یعنی با اشتراک تهی) از این مجموعه چنان یافت که مجموع اعضای یکی با مجموع اعضای دیگری برابر باشد.

موفق باشید.

ارسال متن: شنبه 22 اردیبهشت 1386

mir@
13-05-2007, 11:25
با سلام

مجموعه ی دلخواه 10 عضوی از اعداد طبیعی کوچکتر از 100 را در نظر بگیرید. ثابت کنید که می توان دو زیر مجموعه ی ناتهی مجزا ( یعنی با اشتراک تهی) از این مجموعه چنان یافت که مجموع اعضای یکی با مجموع اعضای دیگری برابر باشد.

موفق باشید.

ارسال متن: شنبه 22 اردیبهشت 1386

درود

فرض کنیم S یک زیرمجموعه 10 عضوی دلخواه از اعداد طبیعی کوچک تر از 100 باشد.

در بدترین حالت، فرض کنیم S به صورت زیر باشد:


S={90,91,92,93,94,95,96,97,98,99}i

مجموع اعداد بالا مساوی است با 945.

اما تمام زیر مجموعه های S برابر است با 1024=10^2 که اگر زیرمجموعه تهی را حذف کنیم، 1023 زیر مجموعه از S خواهیم داشت.

از آنجا که مجموع اعضاء تمام زیرمجموعه های قابل تصور از S یک عدد صحیح مثبت و کوچکتر از یا مساوی با 945 است و 945<1023، لذا طبق اصل «لانه کبوتری» حتماً دو زیر مجموعه یافت می شود که مجموع اعضایشان با هم برابر است. ▲

پاسخ به شبهه مطرح شده توسط Iron:

در صورتی که دو زیر مجموعه اشتراک داشته باشند با هم، مشکل مهمی نیست. چرا که می توان اعضاء مشترک را از هر دو حذف کرد و اگر مجموع تا کنون مقدار M بود حالا برابر خواهد شد با M-m که m منظور مجموع اعضاء مشترک است.

Iron
14-05-2007, 12:15
امیر جان! اشتراک اون دوتا زیر مجموعه باید تهی باشه ها! [ برای مشاهده لینک ، لطفا با نام کاربری خود وارد شوید یا ثبت نام کنید ]

mir@
14-05-2007, 13:05
امیر جان! اشتراک اون دوتا زیر مجموعه باید تهی باشه ها! [ برای مشاهده لینک ، لطفا با نام کاربری خود وارد شوید یا ثبت نام کنید ]

Iron جان، تو هم خوب به ما گیر دادیا.

ولی کامنت خوبی بود. لختی درنگ کن تا بیندیشم.
ـــــــــــــــ
اصلاح گردید. رجاء واثق دارم که اشکال مرتفع گردیده باشد.

Iron
15-05-2007, 03:03
اصلاح گردید. رجاء واثق دارم که اشکال مرتفع گردیده باشد.

برادر جان! پاسخ کوبنده بود!

aghaham
17-05-2007, 13:37
ضرايب لاگرانژ و اكسترمم مقيد,ميشه در مورد اينهاتوضيح بديديا مقاله اي برام بزاريد

Iron
18-05-2007, 01:49
سلام [ برای مشاهده لینک ، لطفا با نام کاربری خود وارد شوید یا ثبت نام کنید ]

مطلب مربوط به این موضوع در کتابها یافت میشه.
اما در مورد این روش:
فرض کنید یک تابع f داشته باشید بر حسب چند متغیر مستقل. حالا اگه بخواید اکستریممهای اونو پیدا کنید، باید مشتقات جزیی اونو بگیرید و برابر با صفر قرار بدید. سپس معادلات بدست اومده رو حل کنید.

اما اگر تابعی با چند متغیر وابسته داشته باشید، باید از ضریب لاگرانژ استفاده کرد. در این حالت n متغیر داریم و m معادله قید بین این متغیر ها وجود دارد. این معادلات بصورت زیر می باشند:

[ برای مشاهده لینک ، لطفا با نام کاربری خود وارد شوید یا ثبت نام کنید ] ([ برای مشاهده لینک ، لطفا با نام کاربری خود وارد شوید یا ثبت نام کنید ])

حال معادلات مربوط به اکستریمم تابع بصورت زیر نوشته می شوند:

[ برای مشاهده لینک ، لطفا با نام کاربری خود وارد شوید یا ثبت نام کنید ] ([ برای مشاهده لینک ، لطفا با نام کاربری خود وارد شوید یا ثبت نام کنید ])

همانطورکه مشخصه m متغیر لاندا اضافه شده است و در نهایت m+n متغیر داریم. معادلات بالا بهمراه معادلات قید، m+n معادله را تشکیل می دهند. و با حل معادلات، مقادیر اکستریمم بدست می آیند.

pp8khat
18-05-2007, 15:35
سلام.لطفاً به این سوالات زود پاسخ بدید آخه فردا امتحان ریاضی دارم.
الف __a+b.c=?
ب __ (a+b) ضرب درc =?

Alireza_Shafaei_PCworld
18-05-2007, 15:39
سلام.لطفاً به این سوالات زود پاسخ بدید آخه فردا امتحان ریاضی دارم.
الف __a+b.c=?
ب __ (a+b) ضرب درc =?

میشه بیشتر توضیح بدی؟

pp8khat
18-05-2007, 15:48
میشه بیشتر توضیح بدی؟
آره.اون aوbوc رو عدد فرض کن مثلاً آیا a+b.c=a+bc یا ac+bc

pp8khat
18-05-2007, 16:12
کسی نیست به من جواب بده!!

mir@
18-05-2007, 18:56
[ برای مشاهده لینک ، لطفا با نام کاربری خود وارد شوید یا ثبت نام کنید ]

sohreh
19-05-2007, 20:15
[quote=pp8khat;1099849]کسی نیست به من جواب بده!![/q


salam.tooye a+b.c avval b ro dar c zarb mikoni bad hasel ro ba a jam mikoni.amma tooye (a+b).c avval a ro ba b jam mikoni bad hasel ro dar c zarb mikoni

Vmusic
19-05-2007, 20:56
با سلام خدمت شما !!!

لطفا فارسی بنویسید تا به یکی از قوانین انجمن عمل کرده باشید !!

ممنون موفق باشید !!

behnam_roli
20-05-2007, 07:51
با عرض سلام من یه مشکلی دارم که نمیدونم باید اینجا مطرح کنم یانه.
فقط میخوام یه توضیح کوچیک در رابطه با روابط مثلثاتی و اینکه اینا از کجا میان بدید(در حد دوم دبیرستان)

pp8khat
20-05-2007, 11:04
[quote=pp8khat;1099849]کسی نیست به من جواب بده!![/q


salam.tooye a+b.c avval b ro dar c zarb mikoni bad hasel ro ba a jam mikoni.amma tooye (a+b).c avval a ro ba b jam mikoni bad hasel ro dar c zarb mikoni

دستت درد نکنه:46: .اما نوش دارو پس از مرگ سهراب
امتحان ریاضی رو دادیم پدرمون دراومد:41:

sohreh
20-05-2007, 16:56
با سلام خدمت شما !!!

لطفا فارسی بنویسید تا به یکی از قوانین انجمن عمل کرده باشید !!

ممنون موفق باشید !!


واقعا" معذرت می خوام.:41: :11:

sohreh
20-05-2007, 17:13
با عرض سلام من یه مشکلی دارم که نمیدونم باید اینجا مطرح کنم یانه.
فقط میخوام یه توضیح کوچیک در رابطه با روابط مثلثاتی و اینکه اینا از کجا میان بدید(در حد دوم دبیرستان)


اگر در دستگاه مختصات دایره ای به مرکز مبدا مختصات و شعاع یک واحد را در نظر بگیریم ,یک دایره ی مثلثاتی رو در نظر گرفته یم.حالا تصور کنید توی این دایره می تونیم شعاعی رو که روی طرف مثبت محور x هاست رو بلند کنیم و در جهت خلاف عقربه های ساعت(یا حتی هم جهت) بچرخونیم.در این صورت بین این شعاع و طرف مثبت x ها یک زاویه ایجاد میشه.حالا اگر از نقطه ای که این شعاع دایره رو قطع می کنه به محور x ها عمود کنیم ,فاصله ی صفر تا پای عمود کسینوس این زاویه ست.و اگر از این نقطه به محور y ها عمود کنیم فاصله صفر تا اون نقطه سینوس اون زاویه ست.

اگر به دردتون می خوره بازم بگم.

Alireza_Shafaei_PCworld
20-05-2007, 20:35
با توجه به توضیح بالا برای زاویه 30 درجه شکل زیر را ببین که برات درست کردم!
[ برای مشاهده لینک ، لطفا با نام کاربری خود وارد شوید یا ثبت نام کنید ]

behnam_roli
21-05-2007, 10:29
دست همتون درد نکنه.:31:
:10: :11:

mofidy1
21-05-2007, 22:44
با سلام

مجموعه ی دلخواه 10 عضوی از اعداد طبیعی کوچکتر از 100 را در نظر بگیرید. ثابت کنید که می توان دو زیر مجموعه ی ناتهی مجزا ( یعنی با اشتراک تهی) از این مجموعه چنان یافت که مجموع اعضای یکی با مجموع اعضای دیگری برابر باشد.

موفق باشید.

ارسال متن: شنبه 22 اردیبهشت 1386

با سلام

از mir@ که در پست 793 مساله را حل کردند، تشکر می کنم. برای دیدن راه حل ایشان به اینجا ([ برای مشاهده لینک ، لطفا با نام کاربری خود وارد شوید یا ثبت نام کنید ]) مراجعه فرمایید.

موفق باشید.

ارسال متن: دوشنبه 31 اردیبهشت 1386

mofidy1
21-05-2007, 23:06
با سلام

فرض کنید A مجموعه ی اعداد گویای مثبت باشد. همه ی توابعی مانند f را که در شرایط زیر صدق می کنند، بیابید:


[ برای مشاهده لینک ، لطفا با نام کاربری خود وارد شوید یا ثبت نام کنید ]

موفق باشید.

ارسال متن: دوشنبه 31 اردیبهشت 1386

kaakaa
22-05-2007, 01:35
javebe soale avvalet mishe:
2^((n*(n-1)/2)
:5:

kaakaa
22-05-2007, 03:05
جواب مساله هفته ی پنجاه و دوم
بدیهی است که با استقرا می توان اثبات کرد که:
f(x+n)=f(x)+n برای هر n طبیعی
حال فرض کنید x عددی گویا به شکل a/b باشد.داریم:
f(a/b+b^2))^3=f((a/b+b^2)^3)=f((a/b)^3+3*a^2+3*a*b^3+b^6)=f(a/b)^3+3*a^2+3*a*b^3+b^6)

از طرفی داریم
f(a/b+b^2))^3=(f(a/b)+b^2)^3=f(a/b)^3+3*f(a/b)^2*b^2+3*f(a/b)*b^4+b^6)
حال با برابر قرار دادن ایندو رابطه و نیز در نظر گرفتن( u/b=f(a/bخواهیم داشت:
a^2+a*b^2=u^2+u*b^2
پس:
(a^2-u^2=b^2*(u-a
بنابراین:
a=u یا u=a+b^2
پس
f(a/b)=a/b یا f(a/b)=a/b+b
که به راحتی می توان نشان داد اولی درست است.پس تنها جواب f(x)=x یعنی تابع همانی می باشد.:5:

kaakaa
22-05-2007, 03:29
اگه هنوز جوابشو پیدا نکردی بگو برات بنویسمش.:5:

kaakaa
23-05-2007, 02:21
یه سواله خیلی قشنگ::5:
یک n ضلعی منتظم داریم که روی یکی از راس هاش عدد 1 و روی بقیه راس ها عدد 0 نوشته شده است. در هر مرحله یک m ضلعی منتظم که راس های ان روی راس های n ضلعی ابتدایی است انتخاب می کنیم (بدیهی است که باید n بر m بخشپذیر باشدو توجه شود که m عددی ثابت نیست و برای هر m با شرایط گفته شده مجاز به انجام عملیا ت هستیم)و به اعداد روی راس های این m ضلعی انتخاب شده 1 واحد می افزاییم.حال با انجام این عملیات ایا می توانیم تمام اعداد روی راس های n ضلعی را به عددی برابر تبدیل کنیم؟!!!

m_honarmand_j
23-05-2007, 03:10
یعنی چی بدون دستور شرطی . بلا خره باید با یه شرطی کنترل بکنه که عملیات کی باید تموم بشه . نمی شه که یه هو گفت این روز ماله چه ماهی یا چه فصلی ه . اگه پیدا کردید به من هم بگید .

m_honarmand_j
23-05-2007, 12:03
سلام دوست عزیز kaakaa
جواب سوال n ضلعی و در اتاق ترکیبیات دادم . اگه خواستی به اونجا مراجعه کن .

pp8khat
27-05-2007, 21:32
معادله زیر را حل کنید:
x+y+z=5
x+y-z=5
x-y+z=1

Alireza_Shafaei_PCworld
27-05-2007, 21:47
معادله زیر را حل کنید:
x+y+z=5
x+y-z=5
x-y+z=1

!
x-y+z+x+y-z=6
2x=6
x=3
=-=-=-=-=-=-
x+y+z+x+y-z=10
6+2y=10
y=2
=-=-=-=-=-=
=>z=0

Alireza_Shafaei_PCworld
27-05-2007, 21:51
سلام دوست عزیز kaakaa
جواب سوال n ضلعی و در اتاق ترکیبیات دادم . اگه خواستی به اونجا مراجعه کن .

آقا مصطفی این سوال
که جوابش را دادین
میشه بگین منبع سوالش کجا بود؟

pp8khat
28-05-2007, 12:45
!
x-y+z+x+y-z=6
2x=6
x=3
=-=-=-=-=-=-
x+y+z+x+y-z=10
6+2y=10
y=2
=-=-=-=-=-=
=>z=0

سلام.
ماماااااا....:18: :18: :18: جواب درسته!:27:
شما چطوری جلدی اینو حل کردین؟ممکنه روشتونو بنویسین :46: من نشستم 2 ساعت دترمینان اینارو حساب کردم:2:
ولی بگذریم با شعارت حسابی موافقم

pp8khat
28-05-2007, 12:47
سلام.
میشه یکی جزءصحیحو برام توضیح بده.آخه من بدبخت سال اولم و با کلی بدبختی نشستم تابع و حد و مشتقو....قاتی پاتی یاد گرفتم.

Alireza_Shafaei_PCworld
28-05-2007, 12:54
سلام.
ماماااااا....:18: :18: :18: جواب درسته!:27:
شما چطوری جلدی اینو حل کردین؟ممکنه روشتونو بنویسین :46: من نشستم 2 ساعت دترمینان اینارو حساب کردم:2:
ولی بگذریم با شعارت حسابی موافقم

شوخی میکنی؟
یک بار عبارت دوم و سوم را با هم جمع کن!

بعدش جمع عبارت اول و دوم را حساب کن!

یادمه واسه ی امتحان ورودی تیزهوشان راهنمایی از این سوالا بمون میدادن!
اینم یک نمونه ی دیگه! فکر کنم تاحالا با این جور مسئله ها برخورد نکردی!
X+Y=5
X+Z=6
Z+Y=7
X+Y+Z=?

pp8khat
28-05-2007, 18:03
شوخی میکنی؟
یک بار عبارت دوم و سوم را با هم جمع کن!

بعدش جمع عبارت اول و دوم را حساب کن!

یادمه واسه ی امتحان ورودی تیزهوشان راهنمایی از این سوالا بمون میدادن!
اینم یک نمونه ی دیگه! فکر کنم تاحالا با این جور مسئله ها برخورد نکردی!
X+Y=5
X+Z=6
Z+Y=7
X+Y+Z=?

سلام.
متشکرم روشت بدردم خورد.اما به نظر من استفاده از قانون ساروس و دترمینان گیری بهتره چون اگه تعداد مجهولات بیشتر بشه نمیشه همینجوری جمعشون کرد.

M-T-D313
30-05-2007, 07:01
:40: سلام
چون نمی دونستم که چقدر از جزء صحیح را می دونی تعاریف اولیه شو نوشتم دفعه بعد چند خاصیتشو که بلدم برات می نویسم اگه باز هم خواستی یاد بگیری بگو. :40:
به ازای هر عدد حقیقی x، عددی صحیح مانند n وعددی حقیقی مانند p که ( p<1 و p>>0 ) (علامت << یعنی کوچکتر یا مساوی) وجود دارد به طوری که x= n + p
در این صورت عدد صحیح n را جزء صحیح x می نامیم و آنرا با نماد [x] یا(E(x نمایش می دهیم مثلاً اگر
19/2=X باشد
19/0 + 2 = X
2=[X]

pp8khat
30-05-2007, 09:29
:40: سلام
چون نمی دونستم که چقدر از جزء صحیح را می دونی تعاریف اولیه شو نوشتم دفعه بعد چند خاصیتشو که بلدم برات می نویسم اگه باز هم خواستی یاد بگیری بگو. :40:
به ازای هر عدد حقیقی x، عددی صحیح مانند n وعددی حقیقی مانند p که ( p<1 و p>>0 ) (علامت << یعنی کوچکتر یا مساوی) وجود دارد به طوری که x= n + p
در این صورت عدد صحیح n را جزء صحیح x می نامیم و آنرا با نماد [x] یا(E(x نمایش می دهیم مثلاً اگر
19/2=X باشد
19/0 + 2 = X
2=[X]

سلام.:40: :40:
هیچی ازش نمیدونم.
متشکرم.خیلی کمکم کردی.:10: :10:

M-T-D313
04-06-2007, 07:55
سلام
اینم قولی که داده بودم ، چند خواص از جزء صحیح:


1)x-1<[x]≤ x
2)x] ≤ x < [x]+1]
3) n≤[x ] ↔ n≤x
4)x-[x]<1> صفر
5)x]= -[x] ← x є Z
6)x є {R-Z} → [-x]= -[x]-1
7)x є R & n є Z → [x+n]=[x]+n
8)[:40:] [ x є R & n є N → [x/n] = [[x]/n

pp8khat
04-06-2007, 12:22
سلام
اینم قولی که داده بودم ، چند خواص از جزء صحیح:


1)x-1<[x]≤ x
2)x] ≤ x < [x]+1]
3) n≤[x ] ↔ n≤x
4)x-[x]<1> صفر
5)x]= -[x] ← x є Z
6)x є {R-Z} → [-x]= -[x]-1
7)x є R & n є Z → [x+n]=[x]+n
8)[:40:] [ x є R & n є N → [x/n] = [[x]/n


سلام.
خیلی ممنونم اما برای من که فرقی نمیکنه
آخه چیزی نمیفهمم
فکر کنم باید تا سال دوم وایسم.
به هر حال ممنونم:40: :40:

mofidy1
05-06-2007, 12:10
با سلام

فرض کنید A مجموعه ی اعداد گویای مثبت باشد. همه ی توابعی مانند f را که در شرایط زیر صدق می کنند، بیابید:


[ برای مشاهده لینک ، لطفا با نام کاربری خود وارد شوید یا ثبت نام کنید ]

موفق باشید.

ارسال متن: دوشنبه 31 اردیبهشت 1386

با سلام

از kaakaa که در پست 813 به حل مساله پرداختند، متشکرم. برای اینکه کاربران محترم حل مساله را راحت تر مطالعه کنند، روش ایشان را با کمی تغییر ذیلاً توضیح می دهم:

می توان دید برای هر n از اعداد طبیعی داریم:


[ برای مشاهده لینک ، لطفا با نام کاربری خود وارد شوید یا ثبت نام کنید ]

حال اگر a و b دو عدد طبیعی دلخواه باشند داریم:


[ برای مشاهده لینک ، لطفا با نام کاربری خود وارد شوید یا ثبت نام کنید ]

از طرف دیگر


[ برای مشاهده لینک ، لطفا با نام کاربری خود وارد شوید یا ثبت نام کنید ]

حال قرار دهید:


[ برای مشاهده لینک ، لطفا با نام کاربری خود وارد شوید یا ثبت نام کنید ]

لذا


[ برای مشاهده لینک ، لطفا با نام کاربری خود وارد شوید یا ثبت نام کنید ]

بنابر این تابع ذکر شده در مساله، فقط می تواند تابع همانی باشد.

موفق باشید.

ارسال متن: سه شنبه 15 خرداد 1386

توضیح:

دوستان عزیز با حل این مساله، بخش «مساله ی هفته» یکساله شد. انشاء الله با مشورت مدیران و همکاران انجمن علمی قصد داریم شکل و شمایل این بخش را تغییر دهیم تا تعداد بیشتری از دوستان در حل مساله مشارکت کنند.
احتمالا تاپیک جدیدی را به این نام ایجاد خواهیم کرد؛ در آنجا توضیحات مفصلتری را خدمتتان تقدیم می کنم.

اگر مشکلی پیش نیاید اوایل تیر 1386 این بخش را دوباره فعال می کنیم، انشاء الله.

از همه دوستانی که در این یکسال بنده را یاری دادند، صمیمانه تشکر می کنم.

موفق و موید باشید.

majid67110
05-06-2007, 19:44
يك مقاله در رابطه با تاريخ لگاريتم و كاربرد ان در ديروز و امروز
لطفا به اين ادرس ميل كنيد email : majid67110@gmail.com

Alireza_Shafaei_PCworld
06-06-2007, 12:17
یک مسئله خیلی ضایع دارم ولی به روش حل آن خیلی نیاز دارم
اثبات کنید:
[ برای مشاهده لینک ، لطفا با نام کاربری خود وارد شوید یا ثبت نام کنید ]

shadli_m7
07-06-2007, 11:22
سلام دوستان عزیز

من برای توشتن برنا مه ام به زبان c++احتیاج دارم که شما کمکم کنید .

من دنبال الگوریتم محاسبه ی دترمینان ماتریس n*n هستم . راستش من فقط تا ماتریس 3*3 رو بلدم بیشتر از اون رو می شه کمکم کنید . ممنونم .

خیلی بهش احتیاج دارم . تشکر

mir@
07-06-2007, 19:43
یک مسئله خیلی ضایع دارم ولی به روش حل آن خیلی نیاز دارم
اثبات کنید:
[ برای مشاهده لینک ، لطفا با نام کاربری خود وارد شوید یا ثبت نام کنید ]

به نظر می رسه x ها باید عدد صحیح باشند در صورت سوال. با این فرض :

صورت مسئله:


[ برای مشاهده لینک ، لطفا با نام کاربری خود وارد شوید یا ثبت نام کنید ] ([ برای مشاهده لینک ، لطفا با نام کاربری خود وارد شوید یا ثبت نام کنید ])


حل با استفاده از استقرا،

بدیهی است برای n=2 درست است. اذا فرض می کنیم عبارت زیر درست باشد:


[ برای مشاهده لینک ، لطفا با نام کاربری خود وارد شوید یا ثبت نام کنید ] ([ برای مشاهده لینک ، لطفا با نام کاربری خود وارد شوید یا ثبت نام کنید ])


بنابراین داریم برای n+1 و با استفاده از معادله بالا:


[ برای مشاهده لینک ، لطفا با نام کاربری خود وارد شوید یا ثبت نام کنید ] ([ برای مشاهده لینک ، لطفا با نام کاربری خود وارد شوید یا ثبت نام کنید ])


بنابراین برای n+1 هم بر قرار است. پس حکم مسئله ثابت می شود.




راه دیگر:

داریم :


[ برای مشاهده لینک ، لطفا با نام کاربری خود وارد شوید یا ثبت نام کنید ] ([ برای مشاهده لینک ، لطفا با نام کاربری خود وارد شوید یا ثبت نام کنید ])


دیده می شود که:


[ برای مشاهده لینک ، لطفا با نام کاربری خود وارد شوید یا ثبت نام کنید ] ([ برای مشاهده لینک ، لطفا با نام کاربری خود وارد شوید یا ثبت نام کنید ])


با جمع عبارت بالا و دوتا بالاتر خواهیم داشت:


[ برای مشاهده لینک ، لطفا با نام کاربری خود وارد شوید یا ثبت نام کنید ] ([ برای مشاهده لینک ، لطفا با نام کاربری خود وارد شوید یا ثبت نام کنید ])

Alireza_Shafaei_PCworld
07-06-2007, 21:22
سلام
خیلی متشکرم
خیلی بدردم خورد
مرسی

mir@
07-06-2007, 22:56
سلام دوستان عزیز

من برای توشتن برنا مه ام به زبان c++احتیاج دارم که شما کمکم کنید .

من دنبال الگوریتم محاسبه ی دترمینان ماتریس n*n هستم . راستش من فقط تا ماتریس 3*3 رو بلدم بیشتر از اون رو می شه کمکم کنید . ممنونم .

خیلی بهش احتیاج دارم . تشکر

سلام
برای محاسبه دترمینان ماتریس های n*n دیگه نباید از روش معمول مانند 3*3 استفاده کرد چرا که اصلاً از لحاظ محاسباتی توجیه زمانی و اقتصادی (!) نداره.

یک روش خیلی خوب و موثر روش تجزیه LU هست که در اون یک ماتریس به دو ماتریس بالامثلثی و پایین مثلثی تجزیه میشه.

حتماً میدونید که دترمینان ماتریس های بالامثلثی یا پایین مثلثی به سادگی برابر است با حاصل ضرب درایه های روی قطر اصلی.

بنابراین دترمینان ماتریس مفروض n*n برابر خواهد شد با حاصلضرب دو ماتریس L و U.


[ برای مشاهده لینک ، لطفا با نام کاربری خود وارد شوید یا ثبت نام کنید ] ([ برای مشاهده لینک ، لطفا با نام کاربری خود وارد شوید یا ثبت نام کنید ])

الگوریتم تجزیه LU به سادگی قابل پیاده سازیه. برای اطلاعات بیشتر می تونید به کتاب های محاسبات عددی رجوع کنید.

برای یک سری توضیحات اختصاری و خیلی خوب راجع به «محاسبه دترمینان» و «روش تجزیه LU» به اینجا ([ برای مشاهده لینک ، لطفا با نام کاربری خود وارد شوید یا ثبت نام کنید ]) و اونجا ([ برای مشاهده لینک ، لطفا با نام کاربری خود وارد شوید یا ثبت نام کنید ])(!) مراجعه کنید.
:11:

mir@
07-06-2007, 23:07
من اینو پیدا کردم ولی خودم امتحان نکردم.


برای مشاهده محتوا ، لطفا وارد شوید یا ثبت نام کنید

mir@
07-06-2007, 23:09
اینم مثل قبلی

برای مشاهده محتوا ، لطفا وارد شوید یا ثبت نام کنید

mir@
07-06-2007, 23:10
این رو هم من امتحان نکرده ام.

برای مشاهده محتوا ، لطفا وارد شوید یا ثبت نام کنید

shadli_m7
08-06-2007, 10:42
ممنون امیر خان

جامع و کامل بود تشکر

eh_mn
11-06-2007, 10:44
دوستی که این سوال رو مطرح کرده بود ادعا می کرد تا حالا حل نشده .
فرض کنیم
[ برای مشاهده لینک ، لطفا با نام کاربری خود وارد شوید یا ثبت نام کنید ]
ثابت کنید
[ برای مشاهده لینک ، لطفا با نام کاربری خود وارد شوید یا ثبت نام کنید ]
که در آن منظور از توان m-ام f ، ترکیب f با خودش به تعداد m بار است.

eh_mn
12-06-2007, 01:45
لطفا در مورد سوال زير راهنمايي كنيد.

فرض كنيم G يك گروه متناهي از مرتبه n باشد و d|n. در اينصورت تعداد اعضاي G كه مرتبه آن‌ها عدد d را مي‌شمارد (با احتساب عنصر هماني) ، مضربي از d است.

با تشكر.

mofidy1
12-06-2007, 08:58
دوستی که این سوال رو مطرح کرده بود ادعا می کرد تا حالا حل نشده .
فرض کنیم
[ برای مشاهده لینک ، لطفا با نام کاربری خود وارد شوید یا ثبت نام کنید ]
ثابت کنید
[ برای مشاهده لینک ، لطفا با نام کاربری خود وارد شوید یا ثبت نام کنید ]
که در آن منظور از توان m-ام f ، ترکیب f با خودش به تعداد m بار است.

با سلام

بله دوست عزیز، این مساله به «حدس کولاتز» (collatz conjecture) معروف است و ظاهراً حدود 70 سال است که بی جواب مانده. دوستان عزیز بد نیست که با میپل (یا نرم افزار های دیگر) برنامه ای برای آن بنویسید و آنرا بیازمایید. اردوش - ریاضیدان فقید - در باره ی این مساله گقته بود که ریاضیات امروزی برای حل آن آمادگی لازم را ندارد. او در سال 1958 پانصد دلار برای حل کننده آن جایزه تعیین کرد!! در لینک زیر مقاله ی زیبایی از این حدس را خواهید دید:

[ برای مشاهده لینک ، لطفا با نام کاربری خود وارد شوید یا ثبت نام کنید ]

موفق باشید.

22 اردیبهشت 1386

aahhmad
12-06-2007, 14:41
با عرض سلام و خسته نباشيد
اولا دوست من بهتر بود شما قوانين مربوط به سايت رو مي خوندي تا متوجه بشي که در اين سايت حتما بايد فارسي تايپ کني
ثانيا ورود شما رو به پي سي ورلد تبريک مي گوييم
ثالثا برادر بزرگوار جناب آقاي مفيدي ( شايد هم خانم ) اگر در زمينه رياضيات احتياج به کمک داشتيد من در خدمتم
مرسسسسسسسسسيييييييي

دوست عزيز شما در درس رياضي به من كمك ميكنيد.ممنونم:11: :11: :11:

aahhmad
12-06-2007, 15:13
:20: من نمونه سوال رشته رياضي ميخواهم باي كنكور باتشكرaa

mir@
12-06-2007, 21:08
:20: من نمونه سوال رشته رياضي ميخواهم باي كنكور باتشكرaa

به عدد موهای سر بنده و شما کتاب در زمینه ریاضیات کنکور نوشته شده که خیلیاش هم خوبه.

چه نیازی هست که اینجا دنبالش بگردید؟

وقتت رو اینجا تلف نکن دوست عزیز. بر فرض اگر پیدا هم بشه به درد نمی خوره که ... :31:

برو چندتا کتاب خوب بگیر بشین با دقت و حوصله بخون. ... پاشو .... پاشو دیگه ...

:11:

pp8khat
14-06-2007, 09:25
سلام.
حالا همون روش محاسبه دترمینان به روش گاوس رو یه جوری می ذارین که ما هم بتونیم ازش استفاده کنیم.

mir@
14-06-2007, 16:59
سلام.
حالا همون روش محاسبه دترمینان به روش گاوس رو یه جوری می ذارین که ما هم بتونیم ازش استفاده کنیم.

محاسبه دترمینان با روش حذف گاوس با قانون های زیر انجام میشه:

1) اگر جای سطر و ستونی عوض شد، دترمینان قرینه می شود.
2) اگر سطری یا ستونی در عددی ضرب شد، دترمینان ماتریس در همان عدد ضرب می شود.
3) اگر ضریب سطری با سطری یا ضریب ستونی با ستونی دیگر جمع شود دترمینان تغییر نمی کند.
4) دترمینان ماتریس مثلثی برابر حاصل ضرب درایه های روی قطر اصلی است.

مثال:

ماتریس زیر را در نظر بگیرید:

[ برای مشاهده لینک ، لطفا با نام کاربری خود وارد شوید یا ثبت نام کنید ]

دو برابر ستون سوم را با ستون اول جمع می کنیم تا درایه سطر سوم و ستون اول صفر شود.

[ برای مشاهده لینک ، لطفا با نام کاربری خود وارد شوید یا ثبت نام کنید ]

حالا 5/8 سطر اول را با سط دوم جمع می کنیم تا درایه سطر دوم و ستون اول هم صفر شود.

[ برای مشاهده لینک ، لطفا با نام کاربری خود وارد شوید یا ثبت نام کنید ]

حالا که ماتریس به یک ماتریس بالا مثلثی تبدیل شده، دترمینان آن به سادگی برابر است با:

[ برای مشاهده لینک ، لطفا با نام کاربری خود وارد شوید یا ثبت نام کنید ]

pp8khat
15-06-2007, 18:29
محاسبه دترمینان با روش حذف گاوس با قانون های زیر انجام میشه:

1) اگر جای سطر و ستونی عوض شد، دترمینان قرینه می شود.
2) اگر سطری یا ستونی در عددی ضرب شد، دترمینان ماتریس در همان عدد ضرب می شود.
3) اگر ضریب سطری با سطری یا ضریب ستونی با ستونی دیگر جمع شود دترمینان تغییر نمی کند.
4) دترمینان ماتریس مثلثی برابر حاصل ضرب درایه های روی قطر اصلی است.

مثال:

ماتریس زیر را در نظر بگیرید:

[ برای مشاهده لینک ، لطفا با نام کاربری خود وارد شوید یا ثبت نام کنید ]

دو برابر ستون سوم را با ستون اول جمع می کنیم تا درایه سطر سوم و ستون اول صفر شود.

[ برای مشاهده لینک ، لطفا با نام کاربری خود وارد شوید یا ثبت نام کنید ]

حالا 5/8 سطر اول را با سط دوم جمع می کنیم تا درایه سطر دوم و ستون اول هم صفر شود.

[ برای مشاهده لینک ، لطفا با نام کاربری خود وارد شوید یا ثبت نام کنید ]

حالا که ماتریس به یک ماتریس بالا مثلثی تبدیل شده، دترمینان آن به سادگی برابر است با:

[ برای مشاهده لینک ، لطفا با نام کاربری خود وارد شوید یا ثبت نام کنید ]


عججججججججججبببببب:13: :13: :13:
اما من فکر می کردم که دترمینان جواب معادله های n مجهولی رو میده....پس فایده های دیگه ای هم داره...
متشکرم.

boogiman
17-06-2007, 07:41
یک نفر نیست که تابع رو از اول درس بده راستش من دوم دبیرستان هستم وامسال هم همچین خوب این بخش از کتاب رو نفهمیدم لطفا از اول این بخش خوب همه چی رو توضیح بدین:41: :41:

majid-ar
18-06-2007, 03:05
سلام.

دوستان به من میگید این جور معادله ها رو چه جوری میشه تجزیه کرد:


[ برای مشاهده لینک ، لطفا با نام کاربری خود وارد شوید یا ثبت نام کنید ]

جواب این رو دارم.
روش تجزیه کردن این جور معادله ها رو میخوام.
این رو برای مثال گفتم.

البته همین درجه 3 بسه.دیگه بزرگر رو نمیخوام.


ممنونم:11:

mir@
18-06-2007, 11:38
سلام

در این جور عبارات که برای تجزیه داده میشه معمولاً یکی از ریشه ها، 1، 1-، 2، 2- است. اینها رو توی عبارت امتحان کنید.

هر کدوم عبارت رو صفر کرد، فرض کنید a باشه ، اون وقت عبارت رو بر (x-a) تقسیم کنید. تقسیم جبری (بلدید که؟)

بعد حتماً باقیمانده صفر میشه و عبارت درجه 3 مساوی خواهد بود با حاصل ضرب خارج قسمت و مقسوم علیه.

خارج قسمت حتماً درجه 2 خواهد بود و ممکنه باز هم تجزیه بشه که به سادگی مشخص خواهد شد.

:11:

majid-ar
18-06-2007, 13:56
ممنونم امیر آقا.
حالا فهمیدم چطوری حل میشه.
تقسیم جبری هم خدا رو شکر بلدم.
tnx

soheilsmart
23-06-2007, 17:09
یک نفر نیست که تابع رو از اول درس بده راستش من دوم دبیرستان هستم وامسال هم همچین خوب این بخش از کتاب رو نفهمیدم لطفا از اول این بخش خوب همه چی رو توضیح بدین:41: :41:

اگه بخوای من در خدمتت هستم:10: :10: :10:

pp8khat
24-06-2007, 15:05
سلام.
مجموعه R(مجموعه اعداد حقیقی) با کدام مجموعه زیر هم ارز است؟
الف)n
ب) Z
ج) Q
د) هيدكدام

eh_mn
25-06-2007, 01:50
سلام.
مجموعه R(مجموعه اعداد حقیقی) با کدام مجموعه زیر هم ارز است؟
الف)n
ب) Z
ج) Q
د) هيدكدام

با سلام

مجموعه‌هاي N ، Z و Q هم‌ارزند و هيچكدام با R هم‌ارز نيستند.

pp8khat
25-06-2007, 12:10
سلام.
بچه ها یک سوال سخت:
می تونین تابعی رو مثال بزنید که وقتی متغیرش به سمت بی نهایت میل کنه، تابع به سمت 1 بره؟!:5:

شبنم خانم
29-06-2007, 23:20
سلام.
می خواستم در مورد نحوه ی حل این مسایل کمکم کنید:(اگه در نظرتون مبتدی هست به استادی خودتون ببخشید)

,وقت هایی که sinx مساوی با یک رادیکال تقریبا هست و x رو می خوان.من متاسفانه نمی تونم رادیکال بنوسم اینجا ولی یه چیزی تو مایه های:
یک دوم( رادیکال (دو منهای رادیکال دو)) و امثال اینها با sinx , cosx , tgx ,cotgx .....
مرسی اگه کمکم کنید.
و همچنین یک سوال دیگه: این قبیل سوالات رو هم توضیح بدید:
در مدت 54 دقیقه هر یک از عقربه های ساعت چند رادیان می چرخد؟
2-قطر چرخ جلوی درشکه ای یک متر و قطر چرخ عقب آن 120 سانتیمتر است.وقتی چرخ جلو 70 درجه می چرخدچرخ عقب چند رادیان خواهد گشت؟؟

mir@
30-06-2007, 01:25
سلام.
بچه ها یک سوال سخت:
می تونین تابعی رو مثال بزنید که وقتی متغیرش به سمت بی نهایت میل کنه، تابع به سمت 1 بره؟!:5:

f(x)=sin(x)/x

time prince
30-06-2007, 19:19
هتلي فضايي بي نهايت اتاق دارد كه بي نهايت مسافر هم دارد در همين لحظه يك فضاپيما با بي نهايت مسافر مي رسد چگونه صاحب هتل مي تواند اين مسافران را در هتل جا بدهد؟

mehrdad21
01-07-2007, 03:04
سلام
ممنون از زحمتی که میکشید
. راهنمایی کنید



تمام جوابهاي صحيح معادله زير را بيابيد؟





y^2 = x^3 - 432

mehrdad21
01-07-2007, 05:06
سلام . اول از همه از آقای مفیدی به خاطر تاپیک تکراری عذر میخام چون واقعا یادم رفته بود قبلا تو اینجا اون مطلب رو گذاشتم شرمنده

======

ضرب دو عدد دو رقمی که دهگانشان یکی است و جمع رقمهای یکانشان 10 میشود

1-ابتدا یکان ها را در هم ضرب میکنیم و به عنوان جواب از سمت راست می نویسیم حالا دو رقم دیگر هنوز مانده است

2- عدد یکسان یعنی دهگان را در یک عدد بالاتر از خودش ضرب میکنیم.

مثال :42*48

خب این مثال حاوی شرط ما است یعنی 8+2=10
رقم های دهگان هم که هر دو 4 است پس از روشی که ذکر شد استفاده میکنیم
ابتدا یکانها را در هم ضرب میکنیم 8*2=16
16--
دو رقم سمت راست بدست می آید
سپس دهگان مشترک را در یک عدد بالاتر ضرب میکنیم
4*5=20
این دو رقم را در سمت چپ میگذاریم جواب میشود
2016

time prince
01-07-2007, 08:23
هتلي فضايي بي نهايت اتاق دارد كه بي نهايت مسافر هم دارد در همين لحظه يك فضاپيما با بي نهايت مسافر مي رسد چگونه صاحب هتل مي تواند اين مسافران را در هتل جا بدهد؟
دوستان سوالم خيلي سخته؟

معلممون مي گفت جوابش فقط يك خط توضيحه...

pp8khat
01-07-2007, 16:25
سلام.
می خوام ببینم کی میتونه این سوالو حل کنه غیر از من(چقدر خودمو تحویل گرفتم:31: )

[ برای مشاهده لینک ، لطفا با نام کاربری خود وارد شوید یا ثبت نام کنید ]
حال می کنین؟
سوال از این خفن تر دیده بودید؟!:20:
من حلش کردم:5: ...
:19: این سوال سرکاری نیست!:19:
این سوالو با اندکی تغییر از کتاب تست مبتکران(اندیشه سازان سابق) اینجا گذاشتم.(تبلیغات نشه ولی خداییش کتاب خوبیه:40: :10: )
منتظر جوابم:46:

pp8khat
01-07-2007, 16:27
برای دیدن کامل سوال بالا باید عکسو رو هارد ذخیره کنین...

mir@
01-07-2007, 16:39
جوابش میشه

صفر

چون یه جا اون وسط x-x داریم.

[ برای مشاهده لینک ، لطفا با نام کاربری خود وارد شوید یا ثبت نام کنید ]

FerineFast
01-07-2007, 20:41
1) سوالی برای علاقه مندان به خصوص رده A :
[ برای مشاهده لینک ، لطفا با نام کاربری خود وارد شوید یا ثبت نام کنید ]

FerineFast
01-07-2007, 20:47
سوالی ساده نکنید لطفاً !

[ برای مشاهده لینک ، لطفا با نام کاربری خود وارد شوید یا ثبت نام کنید ]

time prince
02-07-2007, 18:45
هتلي فضايي بي نهايت اتاق دارد كه بي نهايت مسافر هم دارد در همين لحظه يك فضاپيما با بي نهايت مسافر مي رسد چگونه صاحب هتل مي تواند اين مسافران را در هتل جا بدهد؟
يعني هيچ كس بلد نيست؟

rouhallah
04-07-2007, 14:52
يعني هيچ كس بلد نيست؟

با سلام به دوست عزيز
جواب سوال شما رو ميشه با تعاريفي كه در رياضي آمده داد
شما گفتيد هتلي داريم در فضا با بينهايت اتاق و بي نهايت مسافرو سپس فضاپيمايي از راه ميرسه و بينهايت مسافر ديگه تحويل ميده و مشكل جا دادن اين مسافران در اين هتل است
خب اين سوال يك اشكال منطقي داره و اون اينكه اگر بينهايت مسافر درون هتل باشند مسافر ديگري باقي نخواهد ماند كه مشكل جا شدن آنها در هتل را داشته باشيم
حال اگر از اين اشكال منطقي نيز صرف نظر كنيم با توجه به عبارت رياضي زير ميتوان بينهايت مسافر ديگر هم در هتل جا داد و آن عبارت (∞ = ∞ +∞ )
ميباشد
نكته جالب اينكه اگر عبارت ( ∞ = ∞*عدد ) را در نظر بگيريم ميتوان به تعداد دلخواه فضاپيما با بينهايت مسافر به اين هتل فرستاد و همه را در آن جا داد

و جالبتر وقتي هست كه اگر عبارت (∞ = ∞ *∞ ) را در نظر بگيريم آن وقت ميشود بينهايت بار فضاپيما را با بينهايت مسافر به اين هتل فرستاد و همه هم در آن جا شوند
اميدوارم جواب سوالتون رو گرفته باشيد.

time prince
04-07-2007, 16:55
ايول rouhallah جان. كارت حرف نداشت.:40: :11: :10:

mofidy1
08-07-2007, 12:57
با سلام

یکی از دوستان مساله ی زیر رو به طور خصوصی پرسیده بود. فکر کردم اگه عمومی مطرح بشه، شاید بهتر باشه؟

چقدر احتمال داره از بين چهار فرزند دوتاشون تو يه ماه به دنيا بيان؟

ممنون

17 تیر 1386

my friend
12-07-2007, 13:38
لیفا در این مورد راهنمایی کنید:

برای مشاهده محتوا ، لطفا وارد شوید یا ثبت نام کنید

mir@
12-07-2007, 14:24
ببین مبناهای مهم ایناس: 10 و 2 و 8 و 16

تبدیل از مبنای 10 به b

با تقسیمات متوالی انجام میشه. یعنی عدد مورد نظر رو تقسیم بر b می کنیم اگر باز امکان پذیر بود همین طور ادامه میدیم و در پایان از آخر به اول می نویسیم مثل مثال زیر:

در اینجا میخوایم 45 در مبنای 10 رو به مبنای 2 تبدیل کنیم. متوالیاً بر 2 تقسیم می کنیم و بعد از آخرین خارج قسمت می نویسیم و بعد باقیمانده ها رو می نویسیم تا اول:

[ برای مشاهده لینک ، لطفا با نام کاربری خود وارد شوید یا ثبت نام کنید ]

پس 45 در مبنای ده میشه 101101 در مبنای 2.

یه مثال دیگه: تبدیل 45 به مبنای 7 :

[ برای مشاهده لینک ، لطفا با نام کاربری خود وارد شوید یا ثبت نام کنید ]

پس 45 در مبنای ده میشه 63 در مبنای 7.


تبدیل از مبنای b به 10:

هر رقمی در هر عددی یک جایگاهی داره. مثلاً در عدد 514 ، رقم 4 در جایگاه صفر، رقم 1 در جایگاه 1 و رقم 5 در جایگاه 2 هست. اگر این عدد در مبنای b باشه، باید هر رقم رو ضرب در b به توان جایگاه بکنی بعد همه رو با هم جمع کنی.

مثلاً فرض کن 514 در مبنای 5 باشه و بخوای بیاری به مبنای 10. این طوری میشه.
و مثال های دیگه:

[ برای مشاهده لینک ، لطفا با نام کاربری خود وارد شوید یا ثبت نام کنید ]


مبنای 16

اما مبنای 16 هم که حتماً می دونی از عدد 10 تا 15 با حروف نشون داده میشه:
10=A
11=B
12=C
13=D
14=E
15=F


حالا 220 از مبنای ده به 16 می خواهیم تبدیل کنیم:

[ برای مشاهده لینک ، لطفا با نام کاربری خود وارد شوید یا ثبت نام کنید ]

مثال دیگه 2400 از مبنای 10 به 16:

[ برای مشاهده لینک ، لطفا با نام کاربری خود وارد شوید یا ثبت نام کنید ]

حالا از 16 به ده . فرض کن عدد A25FC43B در مبنای 16 رو می خوایم به ده تبدیل کنیم.

[ برای مشاهده لینک ، لطفا با نام کاربری خود وارد شوید یا ثبت نام کنید ]

به جای حروف معادلشون در مبنای ده حساب شده.





برای تبدیل مبناهای 2، 8 و 16 به همدیگه راه جالبی هست که اگه خواستی بهت بگم.

برای تبدیل از هر مبنا به مبنای دیگه میتونی اول به 10 تبدیل کنی و بعد به مبنای دوم.

ضمناً میتونی از ماشین حساب علمی ویندوز هم استفاده کنی.

برای اطلاعات بیشتر:

برای مشاهده محتوا ، لطفا وارد شوید یا ثبت نام کنید

سوالی بود بازم در خدمتیم. :11:

my friend
13-07-2007, 08:37
دستت درد نکنه.
خیلی لطف کردی.

SANR
13-07-2007, 19:57
:10: سلام:

لطفآ به این سوال پاسخ. به همراه توضیحات کامل بدید.واقعآ ممنون میشم.


1=مقدار a را چنان پیدا کنید که نقطه (3a-4و2a+1) که B می باشد روی نیمساز ربع اول و سوم باشد.سپس مختصات نقطه B را به دست آورید.

eh_mn
13-07-2007, 20:55
:10: سلام:

لطفآ به این سوال پاسخ. به همراه توضیحات کامل بدید.واقعآ ممنون میشم.


1=مقدار a را چنان پیدا کنید که نقطه (3a-4و2a+1) که B می باشد روی نیمساز ربع اول و سوم باشد.سپس مختصات نقطه B را به دست آورید.

با سلام.

يك نقطه فقط هنگامي روي نيمساز ربع اول و سوم است كه طول و عرضش باهم برابر باشد. ( چون معادله خط نيمساز بصورت y=x است) بنابراين طول و عرض نقطه B با هم برابرند. يعني 2a+1=3a-4 با حل اين معادله داريم a=5. براي قسمت دوم هم كافيست مقدار a را در مختصات B جايگذاري كنيم با اين حساب داريم

B=(2a+1,3a-4)=(2*5+1,3*5-4)=(11,11)l

موفق باشيد

eh_mn
14-07-2007, 21:07
با سلام.

دوستان لطفا در مورد اين سوال راهنمايي كنيد :

فرض كنيم A يك ماتريس مربعي m*m قطري‌پذير با درايه‌هايي از اعداد مختلط باشد ، طوريكه عدد طبيعي n موجود است كه براي هر ماتريس مربعي ديگر مانند X داريم


[ برای مشاهده لینک ، لطفا با نام کاربری خود وارد شوید یا ثبت نام کنید ]
ثابت كنيد AX=XA.

با تشكر.

SANR
14-07-2007, 22:50
با سلام.

يك نقطه فقط هنگامي روي نيمساز ربع اول و سوم است كه طول و عرضش باهم برابر باشد. ( چون معادله خط نيمساز بصورت y=x است) بنابراين طول و عرض نقطه B با هم برابرند. يعني 2a+1=3a-4 با حل اين معادله داريم a=5. براي قسمت دوم هم كافيست مقدار a را در مختصات B جايگذاري كنيم با اين حساب داريم

B=(2a+1,3a-4)=(2*5+1,3*5-4)=(11,11)l

موفق باشيد


دستت درد نکنه.یک سوال؟اگه جای 5 هر عددی زدیم مشکلی نیست؟


راستی روی تمام نیم سازهای دیگه اگه ممکنه توضیح بدید.


مثلآ دوم و چهارم.اول و دوم.سوم و چهارم.

SANR
14-07-2007, 22:52
ببخشید ها چون من خنگ هم هستم.واقعآ مدیون شما میشم اگه کمکم کنید.

eh_mn
14-07-2007, 23:07
دستت درد نکنه.یک سوال؟اگه جای 5 هر عددی زدیم مشکلی نیست؟


راستی روی تمام نیم سازهای دیگه اگه ممکنه توضیح بدید.


مثلآ دوم و چهارم.اول و دوم.سوم و چهارم.

با سلام.

1- منظورتون از اينكه " بجاي 5 هر عددي بزنيم " چيه ؟ توجه كنيد كه عدد 5 از حل يك معادله بدست اومده.

2- دوست عزيز در مورد سوال دومت اگه ممكنه به من بگو كه معادله خط نيمساز اول و دوم يا معادله خط نيمساز سوم و چهارم چيه ؟

موفق باشيد.

SANR
15-07-2007, 23:36
با سلام.

1- منظورتون از اينكه " بجاي 5 هر عددي بزنيم " چيه ؟ توجه كنيد كه عدد 5 از حل يك معادله بدست اومده.

ببخشید ها چطوری به دستش اوردید؟

2- دوست عزيز در مورد سوال دومت اگه ممكنه به من بگو كه معادله خط نيمساز اول و دوم يا معادله خط نيمساز سوم و چهارم چيه ؟
منظورم اینه که همین سوال رو یه جور دیگه بیاد.


راستی خوشحال میشم یک سوال هم آخر کار بپرسید تا ببینم می تونم حلش کنم یا نه.

موفق باشيد.

mir@
16-07-2007, 23:21
ببخشید ها چطوری به دستش اوردید؟
منظورم اینه که همین سوال رو یه جور دیگه بیاد.


راستی خوشحال میشم یک سوال هم آخر کار بپرسید تا ببینم می تونم حلش کنم یا نه.

موفق باشيد.


ببين دوست عزيز،

اگر نقطه اي بخواد روي نيمساز ربع اول و سوم باشه، بايد x و y اون با هم مساوي باشه، بنابراين چون نقطه شما اينه:
B(2a+1,3a-4)c بنابراين بايد مختصات اول و دومش رو مساوي بذاريم. يعني:
2a+1=3a-4
اگر اين معادله رو حل كني جواب ميشه a=5

اما ما فقط دو جور نيمساز داريم، نيمساز ربع اول و سوم و نيمساز ربع دوم و چهارم. به شكل زير توجه كنيد:

[ برای مشاهده لینک ، لطفا با نام کاربری خود وارد شوید یا ثبت نام کنید ]


خط قرمز نيمساز ربع اول و سومه. خط سبز نيمساز ربع دوم و چهارم.

اگر نقطه اي بخواد روي نيمساز ربع دوم و چهارم باشه بايد x وy اون قرينه همديگه باشه.
يعني x=-y

حالا شما اينو بگو:

نقطه A(2a+4,-3a+6)c روي نيمساز ربع دوم و چهارم هست. a را تعيين كنيد.

pp8khat
16-07-2007, 23:28
ببين دوست عزيز،

اگر نقطه اي بخواد روي نيمساز ربع اول و سوم باشه، بايد x و y اون با هم مساوي باشه، بنابراين چون نقطه شما اينه:
B(2a+1,3a-4)c بنابراين بايد مختصات اول و دومش رو مساوي بذاريم. يعني:
2a+1=3a-4
اگر اين معادله رو حل كني جواب ميشه a=5

اما ما فقط دو جور نيمساز داريم، نيمساز ربع اول و سوم و نيمساز ربع دوم و چهارم. به شكل زير توجه كنيد:

[ برای مشاهده لینک ، لطفا با نام کاربری خود وارد شوید یا ثبت نام کنید ]


خط قرمز نيمساز ربع اول و سومه. خط سبز نيمساز ربع دوم و چهارم.

اگر نقطه اي بخواد روي نيمساز ربع دوم و چهارم باشه بايد x وy اون قرينه همديگه باشه.
يعني x=-y

حالا شما اينو بگو:

نقطه A(2a+4,-3a+6)c روي نيمساز ربع دوم و سوم هست. a را تعيين كنيد.
سلام آقا امیر...
فکر کنم اونجایی که زیرشو خط کشیدمو اشتباه تایپ کرید

mir@
16-07-2007, 23:34
سلام آقا امیر...
فکر کنم اونجایی که زیرشو خط کشیدمو اشتباه تایپ کرید

ممنون از تذكرت. درستش كردم.

SANR
17-07-2007, 14:07
آقا امیر دست شما درد نکنه.الان میرم حلش میکنم.ولی اگه برای نیمساز ربع دوم و چهارم هم یه مثال می زدید عالی بود.

SANR
17-07-2007, 22:23
یه مثال بزنید تا یاد بگیرم واسه ربع دوم و چهارم.

mir@
18-07-2007, 01:13
B( 4a+1, 5a+8)c

a را طوري تعيين كن كه نقطه روي نيمساز دوم و چهارم باشه؟

eh_mn
19-07-2007, 17:09
با سلام.
اگر ممكن است در مورد اين سوال راهنمايي كنيد.




دوستان لطفا در مورد اين سوال راهنمايي كنيد :

فرض كنيم A يك ماتريس مربعي m*m قطري‌پذير با درايه‌هايي از اعداد مختلط باشد ، طوريكه عدد طبيعي n موجود است كه براي هر ماتريس مربعي ديگر مانند X داريم


[ برای مشاهده لینک ، لطفا با نام کاربری خود وارد شوید یا ثبت نام کنید ]
ثابت كنيد AX=XA.

(ماتريس A قطري‌پذير است هرگاه ماتريس معكوس‌پذير P و ماتريس قطري D موجود باشند بطوريكه [ برای مشاهده لینک ، لطفا با نام کاربری خود وارد شوید یا ثبت نام کنید ] )


با تشكر

Sharim
19-07-2007, 17:27
آقا مبارزه میطلبم معمایی که در زیر لینکش رو نوشتم یک معمای ریاضیه جایزه هم داره ببینم چند مرده حلاجید ؟

rouhallah
21-07-2007, 15:13
آقا مبارزه میطلبم معمایی که در زیر لینکش رو نوشتم یک معمای ریاضیه جایزه هم داره ببینم چند مرده حلاجید ؟

با سلام
آيا در اين معما فقط يك چاقو وجود دارد يا همه چاقو دارند و همزمان شروع به كشتن ميكنند؟

Hamid69
21-07-2007, 15:46
یه سوال ساده که نمیدونم جواب داره یا نه؟
چطور عبارت 6x(2)+7x+2 به (2x+1)(3x+2) تبدیل میشه؟
اون دویی که توی پرانتزه توان ایکسه.

پاکر
22-07-2007, 03:37
یه سوال ساده که نمیدونم جواب داره یا نه؟
چطور عبارت 6x(2)+7x+2 به (2x+1)(3x+2) تبدیل میشه؟
اون دویی که توی پرانتزه توان ایکسه.


والا ایشالا اگه قسمت من شد جواب کامل رو بهت می گم
اما اگه نشد که بقیه دوستان هستند
راه حلشم اینه که ابتدا عبارت رو مساوی صفر بذار بعد ریشه ها رو
بدست بیار (از روش دلتا) بدش اگه دو ریشه به ترتیب e و f باشه
و a ضریب x به نمای دو باشه ( دراینجا a=6) تجزیه عبارت به صورت زیر میشه:
a(x-e)(x-f)
با ضرب درست 6 در دو عبارت بالا می تونی به جواب برسی

Feron
22-07-2007, 04:16
[ برای مشاهده لینک ، لطفا با نام کاربری خود وارد شوید یا ثبت نام کنید ]

bache pooldar
24-07-2007, 01:41
با سلام خدمت تمامي اعضا 15 تا سوال تستي داشتم هر كي هر چه قدر مي توني تيك بزنه و يجوري مارو با خبر كنه كل اين سوالات 30 تا بود 15 تا شو حل كردم ولي بقيه اش موندم فوق العاده روش فكر كردم ولي به جايي نرسيدم اصلا هم دوست ندارم كار خودم رو بر دوش ديگران بگذارم اما چاره اي ندارم.

ممنون مي شوم كسي بتونه كمك كنه.

[ برای مشاهده لینک ، لطفا با نام کاربری خود وارد شوید یا ثبت نام کنید ]

Feron
24-07-2007, 05:09
سلام
اين گونه سوالات كه معمولا اولين سوال كنكور سراسري را به خود اختصاص مي دهد و معمولا در كنكور آزاد هم مورد سوال قرار مي گيرد نيازي به حل تشريحي نداشته و راحت ترين راه براي حل آنها تكنيك رد گزينه ميباشد
به عنوان مثال در سوال 17 يه نگاه كوچولو به گزينه ها انداخته و گزينه ي راحت را در صورت سوال امتحان مي كنيم
( گزينه راحت معمولا يك عدد صحيح مثل 0و1و2و3و-1و-2و-3 و... مي باشد.) اگر عدد صفر را در صورت سوال قرار دهيم متوجه مي شويم كه در صورت سوال صدق مي كند پس 0 يكي از جواب هاي نامعادله است.
با نگاه به گزينه هاي ديگر متوجه ميشويم كه در هيچ كدام از گزينه هاي ديگر عدد صفر وجود ندارد
پس گزينه 2 پاسخ صحيح است.
حال با استفاده از اين تكنيك به حل ديگر تست ها بپرداز اگر به مشكلي بر خوردي من در خدمتم

bache pooldar
24-07-2007, 12:43
اين هم يك روش است ولي بسيار زمان بر است

Feron
25-07-2007, 02:18
با سلام مجدد
دوست عزيز اگر شما با استفاده از تكنيك بالا به حل چندين تست ( 60 تست ) بپردازيد به اين نتيجه خواهيد رسيد كه تكنيك بالا تنها روش مطمئن و سريع براي حل اينگونه تست ها مي باشد ( البته اگر شما در مقطع دوم دبيرستان تحصيل ميكنيد اين كندي كاملا طبيعي است ) ما اين روش را بر روي داوطلبان كنكور و المپياد بررسي كرديم و جواب گرفتيم.

pp8khat
25-07-2007, 12:58
با سلام خدمت تمامي اعضا 15 تا سوال تستي داشتم هر كي هر چه قدر مي توني تيك بزنه و يجوري مارو با خبر كنه كل اين سوالات 30 تا بود 15 تا شو حل كردم ولي بقيه اش موندم فوق العاده روش فكر كردم ولي به جايي نرسيدم اصلا هم دوست ندارم كار خودم رو بر دوش ديگران بگذارم اما چاره اي ندارم.

ممنون مي شوم كسي بتونه كمك كنه.

[ برای مشاهده لینک ، لطفا با نام کاربری خود وارد شوید یا ثبت نام کنید ]
سلام.
من حلشون کردم اما بعضا ها رو چون سوال ناخوانا بود نتونستم حل کنم.
بعضی ها هم که بلد نبودم چون هنوز به اونجاها نرسیدیم
به هر حال این هم جوابا با یه کم توضیح
راستی این سوالا رو از کجا ورداشتی؟
1)4
تو این سوال باید توجه کنی که عبارات رو ساده نکنی و مواظب صفر مخرج باشی چون عبارت

تعریف نشده میشه..
2)3
باید عبارات رو تعیین علامت کنی و اشتراکشونو به دست بیاری(اگه تو اشتراک به دست آوردن

مشکل داری تغییرات x در عبارت اول رو بالای یه محور ایکس پریم-او-ایکس نشون بده و

تغییرات x در عبارت دوم رو زیر اون محور نشون بده و جایی رو که دو بار پر رنگ کردی

قبول کن...)
3)1
وقتی داری جمع یا تفریق می کنی (توعبارات این شکلیa<b<c)باید همه رو با چیزی جمع یا

تفریق کنی مثلاً من می خوام a<b<c رو منهای a کنم که میشهa-a<b-a<c-a
4)2
البته توی این سوال یه نکته ی مبهم وجود داره که مربوط می شه به طراح سوال که نتونسته

منظورشو خوب بیان کنه(بر A و B کدامست؟) من که این جوری برداشت کردم که منظور

مولف یعنی اینکه اعدادی که توی مجموعه AوB تعریف کرده رو اگر توی اون عبارت بذاریم

چی میشه...
5)2
توجه کن که هرگز تو این عبارات طرفین وسطین نکنی زیرا بد می شود!(این نکته را از کتاب

اندیشه سازان سابق - مبتکران یاد گرفتم انصافاً کتاب خوبیه و اگه تستای اون رو بزنی

ریاضیدان می شی...)
6)؟
سوال کامل نیست اما من حدس زدم که نامعادله کوچیکتر از صفر بوده و رو این حساب گزینه

1 درسته...
7)1
توجه کن که عباراتی را که درجه x زوجه و بعدش یه عدد مثبته رو نمی خواد تعیین علامت

کنی چون همواره مثبته(همین طور قدر مطلق یا اتحاد های 1 و 2 یا رادیکال ها(فرجه زوج)

....) توام هم یعنی اشتراکشون...
8)؟
گزینه ها 1 و 2 مشابه هستند!من که جواب رو آوردم x<-3 فکر کنم مولف اشتباه کرده...
توجه کن که عبارت زیر رادیکال حتماً باید مثبت باشه تا تعریف شده بشه...
9)1(شک)
این جور سوالا از سوالای مشکل این مبحث حساب می شن...
من هم که کلاس خصوصی می رم معلممون هنوز رو بازه هاست و به قسمت معادله نرسیده

ولی میشه با جا گذاری چند تا مقدار یه چیزایی رو به دست آورد...
اگه1 رو بذاری گزینه 4 رد میشه
اگه1- رو بذاری گزینه 2 رد میشه
حالا می مونه 1 و 4 که من فقط 1- و1 رو پیدا کردم...
البته یکی گفت چون توان x چهار هستش پس 4 تا جواب داره اما من فکر نمیکنم درست باشه
10)؟
این هم از اون سوالاست...من هنوز راه حلشو یاد نگرفتم اما امروز کلاس ریاضی دارم شاید به

این قسمت برسیم..هروقت رسیدیم من هم جواب رو بهت می گم...(شرمنده!)
11)؟
سوال واضح نیست!!این سوالا رو از کجا گرفتی؟من چشمم رو کور کردم نفهمیدم نوشته x-3 یا

x-2 یا x-4!!!اون نوشته بزرگ که رو کاغذ پس زمینه شده چیه؟
12)3
چون بین عبارات ضرب هست پس تک تکشون باید صفر باشه...از اولی جواب میاد 2+, 2-
از دومی جواب میاد 1 پس سه تا جواب داره...(البته به همون دلیل سوال 10 مطمئن نیستم)
13)؟
نیدونم ..به همون دلیل سوال 10!(بازم شرمنده!)
14)؟
نیدونم ..به همون دلیل سوال 10!(به توان n شرمنده!)
15)3
چیه؟تعجب کردی این یکی رو تونستم حل کنم؟بذار بهت بگم که با هزار تا بدبختی حلش کردم!

مشابه اش تو کتاب اندیشه سازان سابق-مبتکران بود!باید طرفین رو به توان 2 می رسوندی...
16)3
مواظب باش طرفین وسطین نکنی!!!
17)4
طرفین رو به توان 2 برسونی حل میشه!!

pp8khat
25-07-2007, 13:06
با سلام مجدد
دوست عزيز اگر شما با استفاده از تكنيك بالا به حل چندين تست ( 60 تست ) بپردازيد به اين نتيجه خواهيد رسيد كه تكنيك بالا تنها روش مطمئن و سريع براي حل اينگونه تست ها مي باشد ( البته اگر شما در مقطع دوم دبيرستان تحصيل ميكنيد اين كندي كاملا طبيعي است ) ما اين روش را بر روي داوطلبان كنكور و المپياد بررسي كرديم و جواب گرفتيم.

سلام.
ببخشید،جسارت نباشه می تونم بپرسم شما واسه چه موسسه ای کار میکنین؟
(اگه نمی خواین مجبور نیستید جواب بدید)
روشی هم که ذکر کردید تا حدی می تونه موثر باشه اما ریسکش به کنکور نمی ارزه چون طراح های سوال یه کاری می کنن که نشه چند تا عدد جادویی(همون 1و2و3و0و1-و2-و3-)رو جاگذاری کرد.
مثلاً شرط هایی مثل کوچکتر یا مساوی 8/95 یا بزگتر از 5 در رادیکال 3.5 میذارن که پدر آدم در میاد تا تازه بفهمیم اون شرط چیه...
البته من این روش رو تو تست های مرآت تست کردم و جواب گرفتم...
درباره المپیاد هم که اصلاً مگه حاصل عبارت می دن؟
یه سوال16 خطی میدن میگن کدوم درست ممکنه باشه:41::18:

Arash_XL7710i_207
25-07-2007, 14:02
جواب اين چجوريه؟؟ (مربوط به آناليز تركيبي و احتمال هست)
سوال : از رابطه C(n,n-2)=120 عدد n كدام است؟

18 16 15 12

bache pooldar
25-07-2007, 15:50
از همه ممنونم خيلي كمكم كرديد

Feron
25-07-2007, 17:26
با سلام
دوست گرامي بنده براي هيچ موسسه اي كار نمي كنم اما از آنجا كه آموزش و پرورش معمولا براي آموزش داوطلبان المپياد به دانشجويان رشته ي دكتري متوسل مي شود ما هم افتخار آموزش اين گونه افراد را داشته ايم
واما در مورد قسمت دوم فرمايشات شما بايد بگم درست است كه سوالات المپياد ها 16 خط است اما در اكثر اينگونه سوالات در آخرين قسمت به يك معادله يا نامعادله مي رسيم كه باز هم بايد به تكنيك بالا متوسل شويم
ضمنا اگر سوالات كنكور سراسري سال هاي گذشته را بررسي كني متوجه مي شوي كه جواب اكثر آن ها عدد صحيح است.

pp8khat
25-07-2007, 19:07
با سلام
دوست گرامي بنده براي هيچ موسسه اي كار نمي كنم اما از آنجا كه آموزش و پرورش معمولا براي آموزش داوطلبان المپياد به دانشجويان رشته ي دكتري متوسل مي شود ما هم افتخار آموزش اين گونه افراد را داشته ايم
واما در مورد قسمت دوم فرمايشات شما بايد بگم درست است كه سوالات المپياد ها 16 خط است اما در اكثر اينگونه سوالات در آخرين قسمت به يك معادله يا نامعادله مي رسيم كه باز هم بايد به تكنيك بالا متوسل شويم
ضمنا اگر سوالات كنكور سراسري سال هاي گذشته را بررسي كني متوجه مي شوي كه جواب اكثر آن ها عدد صحيح است.

سلام.میشه بگین که تو کدوم شهر کار می کنین؟
آخه من داوطلب المپیادم!
اما نه ریاضی بلکه فیزیک!
اما تو شهر ما از این برنامه ها نیست!
میشه بگین کسی که دکترای فیزیک داره با شما همکاره و اسم شهری رو که توش کار می کنه بدین؟

mofidy1
25-07-2007, 20:38
سلام.میشه بگین که تو کدوم شهر کار می کنین؟
آخه من داوطلب المپیادم!
اما نه ریاضی بلکه فیزیک!
اما تو شهر ما از این برنامه ها نیست!
میشه بگین کسی که دکترای فیزیک داره با شما همکاره و اسم شهری رو که توش کار می کنه بدین؟

با سلام

دوست عزیز اینجا جای پرسیدن سوالات علمی است نه آشنایی با یکدیگر!! این سوالات را می توانید در پپامهای خصصوصی از یکدیگر بپرسید.

خواهش می کنم رعایت کنید.

toadstool
26-07-2007, 00:17
جواب اين چجوريه؟؟ (مربوط به آناليز تركيبي و احتمال هست)
سوال : از رابطه C(n,n-2)=120 عدد n كدام است؟

18 16 15 12



[ برای مشاهده لینک ، لطفا با نام کاربری خود وارد شوید یا ثبت نام کنید ]

پاکر
27-07-2007, 01:25
دوستان این یه سوال جالب به کمی خلاقیت نیاز داره
حل این سوال رو به دوستان دبیرستانی توصیه می کنم:
تابع زیر را در نظر بگیرید:

[ برای مشاهده لینک ، لطفا با نام کاربری خود وارد شوید یا ثبت نام کنید ]

1.با استفاده از تعریف نشان دهید که تابع فوق یک به یک است.
2.ضابطه تابع معکوس را بیابید(جواب این قسمت در یکی از تاپیکهای انجمن آمده است.)

pp8khat
29-07-2007, 09:00
سلام.
بچه ها منم یه سوال از توابع داشتم:46:لطفاً دلیلشم بگین:31:
کدامیک از توابع زیر در R،رابطه یک تابع نیست؟
[ برای مشاهده لینک ، لطفا با نام کاربری خود وارد شوید یا ثبت نام کنید ]

mir@
29-07-2007, 11:56
سلام.
بچه ها منم یه سوال از توابع داشتم:46:لطفاً دلیلشم بگین:31:
کدامیک از توابع زیر در R،رابطه یک تابع نیست؟
[ برای مشاهده لینک ، لطفا با نام کاربری خود وارد شوید یا ثبت نام کنید ]
گزینه اول

چون اگر به x مثلاً بدی 0 برای y دو تا جواب به دست میاد: 2+ و 2-

eh_mn
29-07-2007, 16:44
با سلام.

دوستان لطفا در مورد اين سوال راهنمايي كنيد :

فرض كنيم A يك ماتريس مربعي m*m قطري‌پذير با درايه‌هايي از اعداد مختلط باشد ، طوريكه عدد طبيعي n موجود است كه براي هر ماتريس مربعي ديگر مانند X داريم


[ برای مشاهده لینک ، لطفا با نام کاربری خود وارد شوید یا ثبت نام کنید ]
ثابت كنيد AX=XA.

با تشكر.

F1.....F1.......F1........F1....!!!!!

pp8khat
29-07-2007, 21:06
گزینه اول

چون اگر به x مثلاً بدی 0 برای y دو تا جواب به دست میاد: 2+ و 2-

مگه همیشه x مغیره؟
یعنی نمیشه y متغیر باشه و x تابعی از اون..
به نظر شما سوال اشکال نداره...
ما تو فیزیک تابع x(t) داریم.x تابعی از t هستش...

mir@
29-07-2007, 22:51
بله ... اگر بخواهيم خيلي سخت بگيريم ميشه گفت ايراد داره.

ولي خوب معمولاً y تابعي است از x ديگه.

rouhallah
31-07-2007, 11:38
با سلام
يك سوال برام پيش اومده و اون اينكه شنديم و ديدم كه اعداد اول اعداد فرد و مثبت هستند
چه دليلي وجود داره كه اعداد اول منفي وجود نداره
به نظرم نبايد به خاطر اينكه اعداد منفي متناظر اعداد اول مثبت به جز خودشون و يك بر مثبت خودشون و مثبت يك هم بخشپذيزند و اينطوري بر چهار عدد بخش پذير ميشند رو بهونه قرار داد و گفت اعداد اول منفي وجود ندارد
آيا دليلشون همينه كه گفتم يا چيز ديگه
لطفاً راهنماييم كنيد

پاکر
01-08-2007, 00:37
سلام.
بچه ها منم یه سوال از توابع داشتم:46:لطفاً دلیلشم بگین:31:
کدامیک از توابع زیر در R،رابطه یک تابع نیست؟
[ برای مشاهده لینک ، لطفا با نام کاربری خود وارد شوید یا ثبت نام کنید ]

دوست عزیز سوال اشکالی نداره
بلکه اون چیزی که رایج هستش اینه که x رو متغیر مستقل و y رو متغیر وابسته در نظر میگیرن.
حالا تو این سوال هم همین مورد مد نظر هستش!
اگه هم منظور عکس این موضوع بود کلا سوال غلط میشد!
در مورد بررسی تابع بودن یا نبودن یک عبارت هم به راحتی میشه از تعریف تابع استفاده کرد:
توجه کنید!
تابع: رابطه ای که در آن هیچ دو زوج مرتبی یافت نشود که مولفه های اول آن با هم برابر باشند.
بنابراین ما دو عدد دلخواه x1 و x2 رو انتخاب می کنیم حالا اگر x1=x2 باشه باید بتونیم نتیجه بگیریم که f(x1)=f(x2) تا بگیم که عبارت مورد نظر یک تابع هستش.
در مورد گزینه های 1 و 2 و 4 این موضوع کاملا درست جواب میده(میشه فهمید که عبارت تابع هست یا نه!)
اما به گزینه 3 دقت کنین اگه از تعریف بریم در ظاهر نمیشه چون یه عبارت مثبت رو باید برابر با منفی یه عبارت مثبت بذاریم
اما توجه کنیم که x1وx2 باید حتما جزو دامنه باشن بدیهیه که (0و0) در این عبارت صدق می کنه و کاملا از تعریف هم همین نتیجه حاصل میشه
پس گزینه 3 هم تابع هستش
به صورت بدیهی هم اگه همین موضوع رو در مورد گزینه یک امتحان کنی عزیز می بینی که دو تا y1=+-y2 بدست می یاد که درست نیست و در نتیجه گزینه ی یک تابع نیست.:11:

پاکر
01-08-2007, 01:31
با سلام
يك سوال برام پيش اومده و اون اينكه شنديم و ديدم كه اعداد اول اعداد فرد و مثبت هستند
چه دليلي وجود داره كه اعداد اول منفي وجود نداره
به نظرم نبايد به خاطر اينكه اعداد منفي متناظر اعداد اول مثبت به جز خودشون و يك بر مثبت خودشون و مثبت يك هم بخشپذيزند و اينطوري بر چهار عدد بخش پذير ميشند رو بهونه قرار داد و گفت اعداد اول منفي وجود ندارد
آيا دليلشون همينه كه گفتم يا چيز ديگه
لطفاً راهنماييم كنيد
عزیز تعریف هستش:11:

Feron
02-08-2007, 04:49
با سلام
تنها عدد اول زوج 2 مي باشد
اعداد اول زير مجموعه اي از اعداد طبيعي مي باشند.

toadstool
02-08-2007, 11:58
یک سوال از مثلثات :
کی نمودار sec را روی دایره مثلثاتی بلده





[ برای مشاهده لینک ، لطفا با نام کاربری خود وارد شوید یا ثبت نام کنید ]

pp8khat
02-08-2007, 12:09
بازم سلام بچه ها!
ولی من هنوز هم اون سوال تابع رو که تو امتحان قلمچی داده بودن قبول ندارم...
اصلاً اونوقت ما هنوز به تابع نرسیده بودیم(نامعادله بودیم)و من تابع و زوج مرتب رو سر امتحان یاد گرفتم!!(باور میکنین؟)
البته هنوز هم نرسیدیم...
یه سوال دیگه دارم این علامت نماد چیه؟
دلتای برعکس

pp8khat
02-08-2007, 12:12
این پست اشتباه شد!
پاکش کنید!

پاکر
02-08-2007, 18:17
بازم سلام بچه ها!
ولی من هنوز هم اون سوال تابع رو که تو امتحان قلمچی داده بودن قبول ندارم...
اصلاً اونوقت ما هنوز به تابع نرسیده بودیم(نامعادله بودیم)و من تابع و زوج مرتب رو سر امتحان یاد گرفتم!!(باور میکنین؟)
البته هنوز هم نرسیدیم...
یه سوال دیگه دارم این علامت نماد چیه؟
دلتای برعکس

عزیز فکر کنم منظورت صور عمومی باشه!
این علامت شبیه حرف A برعکس هستش
اگه کاربردش رو هم بخوای در نوشتن تعاریف، قضیه ها و... برای بیان «برای هر» استفاده میشه
عکس کار این علامت رو هم علامت صور وجودی میکنه که مثل حرف E برعکس(حتما E بزرگ!)
هستش که اگه خواستی کاربرد اونم میگم اگه نخواستیم که...:31:

ارادتمند:11:

pp8khat
04-08-2007, 10:22
عزیز فکر کنم منظورت صور عمومی باشه!
این علامت شبیه حرف A برعکس هستش
اگه کاربردش رو هم بخوای در نوشتن تعاریف، قضیه ها و... برای بیان «برای هر» استفاده میشه
عکس کار این علامت رو هم علامت صور وجودی میکنه که مثل حرف E برعکس(حتما E بزرگ!)
هستش که اگه خواستی کاربرد اونم میگم اگه نخواستیم که...:31:

ارادتمند:11:

سلام دوست عزیز
ولی فکر نکنم اون چیزی که شما می گی باشه..
آخه بالاش توان 2 داره!!
اول معادله سای تابع موج الکترون هیدروژنه...
تو زیر نویس کتاب شیمی مبتکران(تالیف مهندس بهمن بازرگانی) دیدمش...
اینم فرمولش:
[ برای مشاهده لینک ، لطفا با نام کاربری خود وارد شوید یا ثبت نام کنید ]
بی خیال..
امروز یا فردا می رم کتاب خونه، شیمی معدنی(تالیف دکتر آقا بزرگ)می خونم ته توشو در میارم...
ممون

mir@
07-08-2007, 22:52
سلام

دلتاي برعكس خونده ميشه: «اپراتور دِل» يا «نابلا»

معمولاً مشتقات پيشرفته اي مثل «كِرل» يا «ديورژانس» رو باهاش نشون ميدن. (ان‌شاءالله در دانشگاه مي‌ خونيد)

اما اگر يك توان 2 هم بالاش داشته باشه ميشه «لاپلاسين» مثلاً عبارت زير

[ برای مشاهده لینک ، لطفا با نام کاربری خود وارد شوید یا ثبت نام کنید ]

را بخوانيد: لاپلاسيــَــنِ u برابر است با صفر

اين آقاي بازرگاني هم مي‌خواسته اظهار فضل كنه كه تو كتاب دبيرستان همچين چيزي رو نوشته :2:

ارادتمند :11:

rafifar
07-08-2007, 23:16
معادله ی زید چند جواب دارد در صورتی که :
الف: Y=1
ب: Y=3
[ برای مشاهده لینک ، لطفا با نام کاربری خود وارد شوید یا ثبت نام کنید ]

لطفا برام حلش کنید. معلممون گفته هر کی حلش کنه پایان ترم از الان بیسته!:27:

pp8khat
08-08-2007, 09:55
سلام

دلتاي برعكس خونده ميشه: «اپراتور دِل» يا «نابلا»

معمولاً مشتقات پيشرفته اي مثل «كِرل» يا «ديورژانس» رو باهاش نشون ميدن. (ان‌شاءالله در دانشگاه مي‌ خونيد)

اما اگر يك توان 2 هم بالاش داشته باشه ميشه «لاپلاسين» مثلاً عبارت زير

[ برای مشاهده لینک ، لطفا با نام کاربری خود وارد شوید یا ثبت نام کنید ]

را بخوانيد: لاپلاسيــَــنِ u برابر است با صفر

اين آقاي بازرگاني هم مي‌خواسته اظهار فضل كنه كه تو كتاب دبيرستان همچين چيزي رو نوشته :2:

ارادتمند :11:

سلام.
آره معلم شیمیمون یه چیزایی می گفت.البته گفت که اینا رو تو شیمی مولتیمر(اگه درست ننوشتم ببخشید)دانشگاه می خونیم...
چقدر سخته ه ه ه ه....
ممنون و متشکر

rafifar
08-08-2007, 15:46
اگه کسی جواب سوال منو می دونه کمکم کنه لطفا خواهش می کنم

toadstool
08-08-2007, 18:56
معادله ی زید چند جواب دارد در صورتی که :
الف: Y=1
ب: Y=3
[ برای مشاهده لینک ، لطفا با نام کاربری خود وارد شوید یا ثبت نام کنید ]

لطفا برام حلش کنید. معلممون گفته هر کی حلش کنه پایان ترم از الان بیسته!:27:

دوست عزيز قسمت الف اش رو حل كردم سعي مي كنم تا امشب جوابش رو اينجا بزارم ( الان يك كم سرم شلوغه)

rafifar
08-08-2007, 19:07
پیشاپیش ازت ممنونم آقا دمت گرم

toadstool
08-08-2007, 20:20
دوست عزيز اين قسمت الف

اميدوارم اشتباه نكرده باشم سطح سوال در حد دوم دبيرستانه

يعني اگه بزرگتر باشي راحت مي فهمي چطور حساب كردم

ولي اگه باز هم سوالي بود در خدمتم

[ برای مشاهده لینک ، لطفا با نام کاربری خود وارد شوید یا ثبت نام کنید ]

rafifar
08-08-2007, 20:27
من دارم می رم اول دبیرستان اما فهمیدم.
برای دومیش چی؟ یعنی مساویه 3؟ اگه واسه اونم بتونید کمکم کنید که هیچی وقتی پایان ترم 20 شدم از ته دل دعاتون می کنم[ برای مشاهده لینک ، لطفا با نام کاربری خود وارد شوید یا ثبت نام کنید ]

mir@
08-08-2007, 21:27
براي Y=3؛
ميدونيم كه تابع كسينوس حداكثر برابر يك هست. پس توان دوش هم همين طور.

حالا وقتي كه مجموع سه جمله 3 ميشه چاره اي نيست جز اينكه هر كدوم يك باشند.

توجه كنيد كه دوره تناوب اين تابع به دليل توان 2 كسينوس برابر پي هست.

لذا بايد x=0 يا x=pi

بنابراين در اين دوره تناوب 2 جواب داريم.

==============================================

اما براي حالت Y=1 پس از مقدار زيادي كار مثلثاتي نهايتاً ميرسيم به sin(4x) cos(3x) = 0
در بازه دوره تناوب داراي 5 جوابه:
30 درجه 45 درجه 135 درجه 90 درجه و 150 درجه.

rafifar
08-08-2007, 21:45
مرسی امیر جون اما اولی رو نفهمیدم :sad:

b59
10-08-2007, 18:06
اگه کسی از دوستان این قضیه رو از طریق قضیه فشردگی میتونه اثبات کنه ممنون میشم کمکم کنه.
اگه لطف کنید جواب رو هم به این آدرس بفرستید ممنون میشم.bahram_2188@yahoo.com
3
x-sinx~1/6x

noia
25-08-2007, 22:48
در کره ای به شعاع R=10 مخروطی محاط نموده ایم . ارتفاع و شعاع قاعده مخروط را چند گیریم که حجم آن Max شود ؟
این سوال از دیشب فکرمو مشغول کرده !
چه جوری حل میشه !؟

پاکر
27-08-2007, 00:32
[ برای مشاهده لینک ، لطفا با نام کاربری خود وارد شوید یا ثبت نام کنید ]

noia
27-08-2007, 13:18
سلام پاکر عزیز , بابا دمت گرم خیلی باحالی ! :10:
خیلی ممنون ! :20::31:

shariatali
27-08-2007, 19:18
در یک ذوزنقه قائم الزاویه اگر اقطار آن بر هم عمود باشند، کدام گزینه همواره درست است (اثبات یا نقض تمام گزینه ها)

1- ارتفاع آن واسطه هندسی بین دو قاعده است.

2- ارتفاع آن واسطه هندسی بین دو قطر است.

3-ارتفاع آن واسطه هندسی بین دو ساق است.

4-ارتفاع آن واسطه هندسی بین یک ساق و یک قاعده است.

mir@
27-08-2007, 21:34
به نظرم چنين ذوزنقه اي بايد مربع باشه.

لذا گزينه اول درسته.

البته مطمئن نيستم.

SuB
27-08-2007, 22:17
دوستان این یه سوال جالب به کمی خلاقیت نیاز داره
حل این سوال رو به دوستان دبیرستانی توصیه می کنم:
تابع زیر را در نظر بگیرید:

[ برای مشاهده لینک ، لطفا با نام کاربری خود وارد شوید یا ثبت نام کنید ]

1.با استفاده از تعریف نشان دهید که تابع فوق یک به یک است.
2.ضابطه تابع معکوس را بیابید(جواب این قسمت در یکی از تاپیکهای انجمن آمده است.)

اینم جواب سوال اولی. فایل PDF با حجم 49 کیلوبایت:

برای مشاهده محتوا ، لطفا وارد شوید یا ثبت نام کنید

تصحیح شد.

pp8khat
28-08-2007, 09:51
به نظرم چنين ذوزنقه اي بايد مربع باشه.

لذا گزينه اول درسته.

البته مطمئن نيستم.

سلام آقا امیر..
این "واسطه ی هندسی" یعنی چی؟
شاید منم بتونم حلش کنم...
ممنون

pp8khat
29-08-2007, 19:30
یه سوال دیگه هم داشتم..
x-x^1/2=1 در مجموعه اعداد حقیقی چند جواب دارد؟
1)0
2)1
3)2
4)بی شمار

پاکر
30-08-2007, 00:41
m
یه سوال دیگه هم داشتم..
x-x^1/2=1 در مجموعه اعداد حقیقی چند جواب دارد؟
1)0
2)1
3)2
4)بی شمار

PP8khat جان بابت اون برنامه ای که تو تاپیک معرفی نرم افزار معرفی کردی، ازت ممنونم:11:
اینم جواب شما
[ برای مشاهده لینک ، لطفا با نام کاربری خود وارد شوید یا ثبت نام کنید ]

mir@
30-08-2007, 06:09
سلام آقا امیر..
این "واسطه ی هندسی" یعنی چی؟
شاید منم بتونم حلش کنم...
ممنون

واسطه هندسي دو عدد يعني جذر حاصل ضرب آنها. مثلاً واسطه هندسي 4 و 9 ميشه 6.


یه سوال دیگه هم داشتم..
x-x^1/2=1 در مجموعه اعداد حقیقی چند جواب دارد؟
1)0
2)1
3)2
4)بی شمار

دو تابع y=x-1 و y=x^0.5 رو در يك محور مختصات بكشيد.

مي بينيد كه فقط در يك نقطه برخورد دارند. پس فقط معادله يك جواب داره.

اشكال جواب آقاي پاكر اينه كه وقتي x^0.5=x-1 نتيجه مي گيريم كه x-1>0 يعني x>1 بنابراين بنابراين اون جوابي كه
كوچكتر از يك ميشه قابل قبول نيست.

در به توان 2 رسوندن بيشتر دقت كنيم:46:

pp8khat
30-08-2007, 10:43
PP8khat جان بابت اون برنامه ای که تو تاپیک معرفی نرم افزار معرفی کردی، ازت ممنونم:11:
اینم جواب شما
[ برای مشاهده لینک ، لطفا با نام کاربری خود وارد شوید یا ثبت نام کنید ]
سلام پاکر جان..
خواهش می کنم..
ولی شما تو جواب به دست آوردن یه اشتباه کوچیکی کردی...
(x2)در دامنه صدق نمی کند...
بنابراین طبق آنچه که آقا امیر گفتن معادله 1 جواب دارد که آن هم تقریباً برابر است با 2.618033989...
ممنون از توجهت

واسطه هندسي دو عدد يعني جذر حاصل ضرب آنها. مثلاً واسطه هندسي 4 و 9 ميشه 6.



دو تابع y=x-1 و y=x^0.5 رو در يك محور مختصات بكشيد.

مي بينيد كه فقط در يك نقطه برخورد دارند. پس فقط معادله يك جواب داره.

اشكال جواب آقاي پاكر اينه كه وقتي x^0.5=x-1 نتيجه مي گيريم كه x-1>0 يعني x>1 بنابراين بنابراين اون جوابي كه
كوچكتر از يك ميشه قابل قبول نيست.

در به توان 2 رسوندن بيشتر دقت كنيم:46:
منم همین جواب رو به دست آوردم اما همه ی دوستام می گن معادله جواب نداره...
دلیل مسخره ای هم دارن!می گن معادله ی گنگ جواب گنگ نداره!!!!
ممنون از راهنمایی هاتون

پاکر
31-08-2007, 00:16
سلام پاکر جان..
خواهش می کنم..
ولی شما تو جواب به دست آوردن یه اشتباه کوچیکی کردی...
(x2)در دامنه صدق نمی کند...
بنابراین طبق آنچه که آقا امیر گفتن معادله 1 جواب دارد که آن هم تقریباً برابر است با 2.618033989...
ممنون از توجهت

منم همین جواب رو به دست آوردم اما همه ی دوستام می گن معادله جواب نداره...
دلیل مسخره ای هم دارن!می گن معادله ی گنگ جواب گنگ نداره!!!!
ممنون از راهنمایی هاتون

آره دوست عزیز من همون ساعت این و تندی حل کردم و به دامنه توجهی نکردم:13:
عجله است دیگه چی کار میشه کرد:31:
مهم همون راه حل کلی هستش که با کمی دقت به جوابم میرسی:11:

rbrj
01-09-2007, 20:59
درود.آیا نرم افزاری جهت درک بهتر مفاهیم هندسه تحلیلی که برای درس ریاضی 2 دانشگاه مفید باشه هست؟ من در تجسم اشکال سه بعدی دچار مشکل هستم. سپاس

SuB
01-09-2007, 23:37
یه سوال دیگه هم داشتم..
x-x^1/2=1 در مجموعه اعداد حقیقی چند جواب دارد؟
1)0
2)1
3)2
4)بی شمار

گزینه 2 درسته.:10:

در مورد دوستات باید بگم یا خیلی خنگ هستند یا دارند سر کارت میزارند.:27: :11:

SuB
01-09-2007, 23:41
در یک ذوزنقه قائم الزاویه اگر اقطار آن بر هم عمود باشند، کدام گزینه همواره درست است (اثبات یا نقض تمام گزینه ها)
1- ارتفاع آن واسطه هندسی بین دو قاعده است.
2- ارتفاع آن واسطه هندسی بین دو قطر است.
3-ارتفاع آن واسطه هندسی بین دو ساق است.
4-ارتفاع آن واسطه هندسی بین یک ساق و یک قاعده است.

من می‌تونم گزینه‌های 2 و 3 و 4 رو رد کنم.:21: اما فعلاً نمی‌تونم ثابت کنم گزینه 1 صحیحه. البته خواستم با روشهای هندسه تحلیلی اثبات کنم ولی یه مشکل :45:کوچیک پیش میاد.:19:

pp8khat
02-09-2007, 15:53
من می‌تونم گزینه‌های 2 و 3 و 4 رو رد کنم.:21: اما فعلاً نمی‌تونم ثابت کنم گزینه 1 صحیحه. البته خواستم با روشهای هندسه تحلیلی اثبات کنم ولی یه مشکل :45:کوچیک پیش میاد.:19:

سلام.
مثل اینکه دوستانم اشتباه می کردن(فقط من تو کلاسمون اون سوال رو درست نوشتم!!)
بقیه یا 2 جواب آوردن یا 0 جواب..(100%زدمااا(اظهار فضل!!!!!))
***
من با هندسه ی تحلیلی گزینه 3 رو میارم!!!
نمی دونم چرا(ولی طبق تعریفی که امیر جان گفت،با محاسبات من گزینه 3 نزدیک ترین است)....

SuB
02-09-2007, 23:06
سلام.
مثل اینکه دوستانم اشتباه می کردن(فقط من تو کلاسمون اون سوال رو درست نوشتم!!)
بقیه یا 2 جواب آوردن یا 0 جواب..(100%زدمااا(اظهار فضل!!!!!))
***
من با هندسه ی تحلیلی گزینه 3 رو میارم!!!
نمی دونم چرا(ولی طبق تعریفی که امیر جان گفت،با محاسبات من گزینه 3 نزدیک ترین است)....
:18::18::18:

'گزینه 2 راحت‌تر از بقیه رد میشه. گزینه 3 هم راحت رد میشه. برای رد گزینه 4 من از نوشتن معادله دایره‌ها و معادله ذوزنقه استفاده کردم.

حالا نمی‌دونم شما چطوری گزینه 1 رو رد یا گزینه 3 رو اثبات می‌کنی!:18::11:

tesla_1369
03-09-2007, 10:27
مساله مطرح شده از قضیه فشردگی به راحتی اثبات می شود این مساله در مورد تانژانت هم صادق است

SuB
04-09-2007, 23:34
در یک ذوزنقه قائم الزاویه اگر اقطار آن بر هم عمود باشند، کدام گزینه همواره درست است (اثبات یا نقض تمام گزینه ها)
1- ارتفاع آن واسطه هندسی بین دو قاعده است.
2- ارتفاع آن واسطه هندسی بین دو قطر است.
3-ارتفاع آن واسطه هندسی بین دو ساق است.
4-ارتفاع آن واسطه هندسی بین یک ساق و یک قاعده است.
الان با اطمینان کامل می‌گم گزینه 1.
می‌تونم مستقیماً با خود روابط هندسی اثبات کنم.

pp8khat
05-09-2007, 10:25
سلام بچه ها!
دوباره من بیسواد با سوالای دوغم اومدم!
دو تا سوال داشتم..
اگر z=x+y تعریف بشه می شه این نتیجه رو گرفت؟؟
f(z)=f(x+y)=f(x)+f(y)a
a بی خوده(برای تایپ بهتر)
یه سوال دیگه..
مثلاً من می خوام الان از x^2+2x+1 مشتق بگیرم..
میشه 2x+2
حالا می خوام ازش انتگرال بگیرم...
می شه x^2+2x...
ولی ثابت C(که در اینجا 1 )رو به من نمیده...
کسی می دونه باید چی کار کرد؟؟
تو یه کتاب قدیمی خوندم می شه ضد مشتق گرفت ولی چیزی حالیم نشد...

SuB
05-09-2007, 20:42
سلام بچه ها!
دوباره من بیسواد با سوالای دوغم اومدم!
دو تا سوال داشتم..
اگر z=x+y تعریف بشه می شه این نتیجه رو گرفت؟؟
f(z)=f(x+y)=f(x)+f(y)a
a بی خوده(برای تایپ بهتر)
یه سوال دیگه..
مثلاً من می خوام الان از x^2+2x+1 مشتق بگیرم..
میشه 2x+2
حالا می خوام ازش انتگرال بگیرم...
می شه x^2+2x...
ولی ثابت C(که در اینجا 1 )رو به من نمیده...
کسی می دونه باید چی کار کرد؟؟
تو یه کتاب قدیمی خوندم می شه ضد مشتق گرفت ولی چیزی حالیم نشد...

در مورد سوال اول باید بگم که در حالت کلی این رابطه برقرار نیست. ولی اگه ( f(x رو برابر x بگیری، اون وقت رابطه برقراره.:5:

انتگرال دو تا داریم. یکی معین و یکی نامعین. انتگرال معین، همون مساحت علامت‌دار زیر نمودار هست ولی انتگرال نامعین مجموعه‌ای از توابع اولیه هست که مشتق‌شون همون تابع تحت انتگرال بشه.
در انتگرال نامعین، به جای اون ثابت، از حرف c استفاده می‌کنند. با توجه به اینکه از انتگرال نامعین بیشتر برای محاسبه انتگرال معین استفاده می شود و با توجه به قضیه دوم اساسی حساب دیفرانسیل و انتگرال نیازی به در نظر گرفتن c برای محاسبه مقدار انتگرال معین نیست.

هیچ راهی وجود نداره که اون ثابت که به شما میده، همون عدد 1 بشه. چون اصلاً تعریف انتگرال معین این رو ایجاب می‌کنه.

بازم سوالی بود در خدمتیم.

pp8khat
05-09-2007, 22:28
در مورد سوال اول باید بگم که در حالت کلی این رابطه برقرار نیست. ولی اگه ( f(x رو برابر x بگیری، اون وقت رابطه برقراره.:5:

انتگرال دو تا داریم. یکی معین و یکی نامعین. انتگرال معین، همون مساحت علامت‌دار زیر نمودار هست ولی انتگرال نامعین مجموعه‌ای از توابع اولیه هست که مشتق‌شون همون تابع تحت انتگرال بشه.
در انتگرال نامعین، به جای اون ثابت، از حرف c استفاده می‌کنند. با توجه به اینکه از انتگرال نامعین بیشتر برای محاسبه انتگرال معین استفاده می شود و با توجه به قضیه دوم اساسی حساب دیفرانسیل و انتگرال نیازی به در نظر گرفتن c برای محاسبه مقدار انتگرال معین نیست.

هیچ راهی وجود نداره که اون ثابت که به شما میده، همون عدد 1 بشه. چون اصلاً تعریف انتگرال معین این رو ایجاب می‌کنه.

بازم سوالی بود در خدمتیم.

ممنون و متشکر...
ای کاش یه راهی وجود داشت...
چون من تا الان هرچی تست حرکت شناسی فیزیک بوده رو تو 2 سوت مشتق می گرفتم!!!
حتی خود مولف ها هم فکر نمی کنم بدونن چنین راهی وجود داره!!!
بازم از توضیحات شما ممنونم

pp8khat
06-09-2007, 16:13
صورت سوال:
محمد88
یکی لطف کنه و به این سوال جواب بده :
تمام اعداد طبیعی را پشت سر هم نوشته ایم.1386 امین رقم این دنباله چیست؟(با راه حل)
الف)9 ب)8 ج)4 د)0 ه)5
__________________
حل:
آلگوریتمی به این شکل می نویسیم:
10 تا می نویسیم و سپس به خط بعد می رویم به این شکل:
1234567891
0111213141
5161718192
0212223242
5262728293
0313233343
.
.
.
می خواهیم بدانیم که 1386 امین رقم این سری چیست؟
خوب تا اینجا که وقت تلف کنی بود!!(البته به تمرکز حواس کمک می کرد!!)
حالا می رسیم به اصل قضیه...(عدد اولی 10+k و عدد دومی رقمی، که به دست می آید)
11-0
21-5
31-0
41-5
51-0
61-5
71-0
81-5
91-0
101-5
111-0
121-5
.
.
.
من برای حل این مسئله یه سری اعداد کوانتومی(!!) ساختم(البته 3 تاست)
1-عدد یکان n امین رقم درخواست شده شماره گروه آن عنصر ببخشید آن رقم را بیان می کند.
2-عدد یکی مانده به یکان(دهگان) n امین رقم درخواست شده مشخص می کند که nامین عدد در خواست
شده در دوره ی 5 ها است یا 0 ها..(1 استثناء می باشد)
3- و بالاخره عدد سوم که مثل عدد کوانتومی Ms (که موقعیت الکترون را کاملاً مشخص می کند) این
عدد آدرس دقیق n امین رقم را به ما می دهد.و این عدد دوره یا تناوب یا سطر آن رقم را بیان می کند
(بابا سوال ریاضی بود یا شیمی!!)
که با فرمول زیر محاسبه می شود:
جمع رقم های nامین رقم درخواست شده=شماره گروه n امین رقم...
برای اینکه موضوع روشن تر شود،مثالی می زنم:
می خواهیم بدانیم 21 امین عدد این دنباله به چیست...
1- عدد یکان 21 یک است،پس این عدد متعلق به گروه اول است.
2- عدد دهگان 21 زوج است پس متعلق به دوره های 5 ها است.(دوره 3 یا 5 یا 7 یا 9 یا 11یا...)
3- مجموع رقم های 21 سه می شود پس این عدد متعلق به گروه دوره سوم است...
با دانستن دوره و گروه 21 در می توانیم بفهمیم که بیست و یکمین رقم این دنباله 5 است.
(البته نیازی به پیدا کردن (2) نبود ولی به فهم بیشتر کمک می کرد.)
خوب حالا می رسیم به 1386...
1- رقم یکان این عدد 6 است،پس متعلق به گروه ششم است.
2- رقم دهگان این عدد زوج است پس به دوره های 5 ها تعلق دارد(دوره 3یا5یا7یا9یا11یا13یا...)
تا اینجا نتیجه می گیریم که رقم آخر 1386 امین این دنباله یکی از اعداد
1
2
3
4
5
6
7
8
9
است.(خسته نباشم!!)
3- مجموع رقم های این عدد 18 می شود پس به تناوب هجدهم تعلق دارد...
به علت کمبود وقت نمی توان آلگوریتم را تا 18 سطر ادامه داد بنابراین از 1 شروع می کنیم به 18 بار
شمردن یعنی هفدهمین بار به 8 میرسیم و هجدهمین بار به 9...
پس رقم آخر 1386امین این دنباله 9 است؛لذا گزینه یک صحیح است...
تمـــــــــــــــــــوم شـــــــــــــــــد!
بازم از این سوالا داشتی بذار...
من که خستگی یه روزم در رفت!

محمد88
06-09-2007, 18:21
3 مضرب 3 بین 2 به توان 100 تا 2 به توان 101 را بگویید؟

SuB
06-09-2007, 20:17
3 مضرب 3 بین 2 به توان 100 تا 2 به توان 101 را بگویید؟

2 به توان 100 بعلاوه 2
2 به توان 100 بعلاوه 5
2 به توان 100 بعلاوه 8
و ....

ali80
06-09-2007, 20:58
2 به توان 100 بعلاوه 2
2 به توان 100 بعلاوه 5
2 به توان 100 بعلاوه 8
و ....
سلام
میشه بگی چجوری بدست آوردی؟؟

ali80
06-09-2007, 21:11
خودم فهمیدم:31:

2^2=3+1
3^2=6+2
4^2=15+1
5^2=30+2
6^2=63+1
.............

===>دو به توان هر عدد زوج - 1 بر 3 بخش پذیر است:5:
===> 100^2-1 بر 3 بخش پذیر است.
با جمع کردن عدد فوق با 3و6و9 عدد های مورد نظر بدست میاد

SuB
06-09-2007, 21:18
خودم فهمیدم:31:

2^2=3+1
3^2=6+2
4^2=15+1
5^2=30+2
6^2=63+1
.............

===>دو به توان هر عدد زوج - 1 بر 3 بخش پذیر است:5:
===> 100^2-1 بر 3 بخش پذیر است.
با جمع کردن عدد فوق با 3و6و9 عدد های مورد نظر بدست میاد






نیاز نبود این همه خودتون رو زجر بدید تا یه قاعده بسازید.:21:

کافی است باقیمانده تقسیم 2 به توان 100 رو بر 3 بدست بیارید. میشه یک. برای اینکه عدد بر 3 بخش‌پذیر باشه، باید 2 تا بهش اضافه کنید. یعنی 2 به توان 100 بعلاوه 2. برای تولید عدد بعدی باید 3 تا بهش اضافه کنید و ....:46:

ali80
06-09-2007, 21:44
نیاز نبود این همه خودتون رو زجر بدید تا یه قاعده بسازید.:21:

کافی است باقیمانده تقسیم 2 به توان 100 رو بر 3 بدست بیارید. میشه یک. برای اینکه عدد بر 3 بخش‌پذیر باشه، باید 2 تا بهش اضافه کنید. یعنی 2 به توان 100 بعلاوه 2. برای تولید عدد بعدی باید 3 تا بهش اضافه کنید و ....:46:
خوب این باقیمانده رو بدست آوردم دیگه
همش یک دقیقه هم نشد.
شما روش دیگری برای بدست آوردن باقیمانده دارید؟؟؟؟؟؟؟؟؟

SuB
07-09-2007, 11:20
خوب این باقیمانده رو بدست آوردم دیگه
همش یک دقیقه هم نشد.
شما روش دیگری برای بدست آوردن باقیمانده دارید؟؟؟؟؟؟؟؟؟

من می دونم 2 تا روش برای محاسبه باقیمانده هست. (البته فکر کنم دو تاش در واقع یکی باشه)
یکی روشی که به کمک هم پیمانگی باقیمانده رو محاسبه می‌کنیم که من این رو بلد نیستم. (توی گسسته پیش‌دانشگاهی می‌خونیم)
یکی دیگه که خودم استفاده می‌کنم و توضیح روشش یکم طولانی هست ولی خیلی جاها باهاش راحت می‌تونی باقیمانده رو محاسبه کنی. با اون روش جواب سوال شما زیر 5 ثانیه داده میشه.:46:

SuB
07-09-2007, 11:22
سلامی دوباره
جواب این تست چیه؟
[ برای مشاهده لینک ، لطفا با نام کاربری خود وارد شوید یا ثبت نام کنید ]

من می‌گم گزینه 3 ولی توی پاسخ نامه نوشته گزینه 4

پاکر
07-09-2007, 21:06
سلامی دوباره
جواب این تست چیه؟
[ برای مشاهده لینک ، لطفا با نام کاربری خود وارد شوید یا ثبت نام کنید ]

من می‌گم گزینه 3 ولی توی پاسخ نامه نوشته گزینه 4

SuB عزیز
اول بابت دعوت نامه باز تشکر میکنم:11:
در مورد همنهشتی درسته توی گسسته می خونید وخیلی هم زیبا هستش و ساده
ولی چون اول باید تعریف همنهشتی رو بخونید من اینجا نمی گم

در مورد سوال تستی هم باید بگم چه خوب بود راه حلت رو میذاشتی چون من که
این جوری حل کردم و جواب هم گزینه 4 بود
خیلی از این خلاصه تر هم میشد بگم ولی گفتم کامل باشه!
احتمالا اشتباه محاسباتی داشتی
[ برای مشاهده لینک ، لطفا با نام کاربری خود وارد شوید یا ثبت نام کنید ]

pp8khat
07-09-2007, 23:20
سلامی دوباره
جواب این تست چیه؟
[ برای مشاهده لینک ، لطفا با نام کاربری خود وارد شوید یا ثبت نام کنید ]

من می‌گم گزینه 3 ولی توی پاسخ نامه نوشته گزینه 4

مگه خبر ندارید سال 79 سخت ترین کنکور رو گرفتن...
ضمناً می شه روش محاسبه باقی مانده توی اون سوال رو توضیح بدید؟
ممنون

Dr_King
08-09-2007, 10:21
سلام خدمت دوستان
دوستان می خواستم بدونم اونی که تو ایران خیلی سریع به سوالات ضرب تقسیم جذر لوگاریتم و ... جواب می ده و کچل هم هست البته پشت سرش مو داره اسمش چیه ؟

pp8khat
08-09-2007, 15:16
سلام.
لطفاً یکی لطف کنه و به این سوال پاسخ تشریحی بده...
سوال:
سه به توان 20 در مبنای 10 چند رقمی است؟
1)20
2)19
3)18
4)17
5)21

mir@
08-09-2007, 16:15
سلام.
لطفاً یکی لطف کنه و به این سوال پاسخ تشریحی بده...
سوال:
سه به توان 20 در مبنای 10 چند رقمی است؟
1)20
2)19
3)18
4)17
5)21

سلام

لگاریتم بلدی عزیزم ؟

لگاریتم 10 میشه 1
لگاریتم 100 میشه 2
لگاریتم 1000 میشه 3
پس اگر عدد p تعداد ارقامش n باشه نتیجتاً


[ برای مشاهده لینک ، لطفا با نام کاربری خود وارد شوید یا ثبت نام کنید ](p)<n

بنابراین اگر میخوای بدونی یک عدد چند رقمیه لگاریتمش رو بگیر و جزء صحیح حاصل رو با یک جمع کن.

بنابراین تعداد ارقام p برابر است با


[ برای مشاهده لینک ، لطفا با نام کاربری خود وارد شوید یا ثبت نام کنید ][\log{p}]+1

لذا تعداد ارقام 3 به توان 20 میشه :


[ برای مشاهده لینک ، لطفا با نام کاربری خود وارد شوید یا ثبت نام کنید ][\log{(3^{20})}]+1=10

و برای چک کردن


[ برای مشاهده لینک ، لطفا با نام کاربری خود وارد شوید یا ثبت نام کنید ]^{20}=3486784401

که می بینید 10 رقمیه.

pp8khat
08-09-2007, 20:38
سلام

لگاریتم بلدی عزیزم ؟

لگاریتم 10 میشه 1
لگاریتم 100 میشه 2
لگاریتم 1000 میشه 3
پس اگر عدد p تعداد ارقامش n باشه نتیجتاً


[ برای مشاهده لینک ، لطفا با نام کاربری خود وارد شوید یا ثبت نام کنید ](p)<n

بنابراین اگر میخوای بدونی یک عدد چند رقمیه لگاریتمش رو بگیر و جزء صحیح حاصل رو با یک جمع کن.

بنابراین تعداد ارقام p برابر است با


[ برای مشاهده لینک ، لطفا با نام کاربری خود وارد شوید یا ثبت نام کنید ][\log{p}]+1

لذا تعداد ارقام 3 به توان 20 میشه :


[ برای مشاهده لینک ، لطفا با نام کاربری خود وارد شوید یا ثبت نام کنید ][\log{(3^{20})}]+1=10

و برای چک کردن


[ برای مشاهده لینک ، لطفا با نام کاربری خود وارد شوید یا ثبت نام کنید ]^{20}=3486784401

که می بینید 10 رقمیه.

اشتباه نمی کنی امیر جان؟
من با ماشین حساب چک کردم شد 19 رقم!!!
من نه لگاریتم بلدم نه جزءصحیح...
اون سوال مال المپیاد ریاضی ویژه سال اول دبیرستان بود سال 85
راه دیگه ای نیست...
شایعاتی درخصوص تقسیم توان بر عددی گویا نفوذ کرده بود...
اما برو بچ سال دوم بلد بودن....
به هر حال ممنونم
___________________________
**********************
+++++++++++++++++++++
نه نه درسته!!!
من صورت سوال رو اشتباه نوشته بودم،سه به توان 40 هستش...
به هر حال مهم یادگرفتن پاسخ تشریحی بود...
ممنون

pp8khat
08-09-2007, 20:53
سلام دوستان...
لطفاً اینا رو هم پاسخ تشریحی بذارید..
ارادتمند شما..
1)چند عدد چهار رقمی abab کم تر از 4000 وجود دارد که دقیقاً 4 مقسوم الیه داشته باشد؟
1)4
2)5
3)6
4)7
5)8
2)به چند طریق می توان 7تا از اعداد 1تا 9 را انتخاب کرد که جمع آن ها بر 3 بخش پذیر باشد؟
1)کمتر از 10
2)10
3)11
4)12
5)بیشتر از 12
3)به چند شکل می توان در جدولی 3*3 سه مهره سفید و سه مهره سیاه را طوری قرار داد که در هر سطر و هر ستون یک مهره سفید و یک مهره سیاه وجود داشته باشد؟
1)6
2)12
3)18
4)24
5)36

4)در اتاقی 5*5 حداکثر چند کاشی 1*3 می توان قرار داد؟
1)5
2)6
3)7
4)8
5)9

با تشکر از توجه تان!:11:

yugioh
08-09-2007, 21:04
سلام سوال 1:
عدد شما:ab+100ab=101ab هست. پس هم 101 وهمab وهم خودش و 1 رو تو مقسوم علیه هاش داره. پس ab باید اول باشه واز 40 هم کمتر. از طرفی a>0 پس ab>9 تعداد اعداد اول بین 10 و40 میشه:گزینه:5)8

yugioh
08-09-2007, 21:08
سوال 2:
مجموع اعداد 1 تا 10 میشه :45 که بر 3 بخش پذیره از طرفی واضحه اگه جمع 7 عدد انتخابی بر 3 بخش پذیر باشه . جمع 2 عدد انتخاب نشده هم باید بر 3 بخش پذیر باشه. از طرفی برای اینکه جمع دو عدد بر 3 بخش پذیر باشه یا هر دو بر 3 بخش پذیرند( 3 حالت) یا باقیمانده هاشون بر 3 1 و2 ( به پیمانه 3 بشن 1و 2) که این هم9 حالت داره ( 3تا برای باقیمانده 2 و 3 تا برای 1) پس در کل میشه: گزینه:4)12

yugioh
08-09-2007, 21:15
سوال 3:
برای سطر اول 6 حالت داریم( 3 برای مهره سیاه و2 برای مهره سفید.) حالا توی سطر دوم مهره سیاه یا زیر سفیده یا زیر خونه خالی. در حالت اول خونه خالی باید زیر مهره سیاه باشه ( چون بنا بر لانه کبوتری اگه نباشه تو سطر آخر یکی از سیاه و سفید توی ستونی که قبلا از اونها داشته میان ( در واقع مهره سیاه و سفید فقط می تونن توی ستون دارای دوتا بدون مهره بیان) اگر هم توی سطر دوم مهره سیاه زیر بدون مهره بیاد برای سفید فقط یک جا می مونه. واضحه که برا سطر آخر هم فقط یه حالت داریم. پس جواب میشه6*2=12(6 سطر اول و 2 سطر دوم) یعنی :گزینه:2)12

yugioh
08-09-2007, 21:20
سوال4:
واضحه جواب حداکثر 8 هست. من راهی می دم که جواب همین بشه( شرمنده عکس نذاشتم عدد های یکسان یک کاشی اند.): پس جواب میشه: گزینه:4)8
1 1 1 2 3
4 4 4 2 3
5 6 * 2 3
5 6 7 7 7
5 6 8 8 8

yugioh
08-09-2007, 21:20
سوال های خوبی بود. مرسی.

SuB
08-09-2007, 21:47
SuB عزیز
اول بابت دعوت نامه باز تشکر میکنم:11:
در مورد همنهشتی درسته توی گسسته می خونید وخیلی هم زیبا هستش و ساده
ولی چون اول باید تعریف همنهشتی رو بخونید من اینجا نمی گم

در مورد سوال تستی هم باید بگم چه خوب بود راه حلت رو میذاشتی چون من که
این جوری حل کردم و جواب هم گزینه 4 بود
خیلی از این خلاصه تر هم میشد بگم ولی گفتم کامل باشه!
احتمالا اشتباه محاسباتی داشتی
[ برای مشاهده لینک ، لطفا با نام کاربری خود وارد شوید یا ثبت نام کنید ]

ممنون
انگار اشتباه محاسباتی بوده.:18:

دیگه زیاد درس خوندن این نتایج رو هم داره.:21:

pp8khat
09-09-2007, 12:12
سلام سوال 1:
عدد شما:ab+100ab=101ab هست. پس هم 101 وهمab وهم خودش و 1 رو تو مقسوم علیه هاش داره. پس ab باید اول باشه واز 40 هم کمتر. از طرفی a>0 پس ab>9 تعداد اعداد اول بین 10 و40 میشه:گزینه:5)8


سوال 2:
مجموع اعداد 1 تا 10 میشه :45 که بر 3 بخش پذیره از طرفی واضحه اگه جمع 7 عدد انتخابی بر 3 بخش پذیر باشه . جمع 2 عدد انتخاب نشده هم باید بر 3 بخش پذیر باشه. از طرفی برای اینکه جمع دو عدد بر 3 بخش پذیر باشه یا هر دو بر 3 بخش پذیرند( 3 حالت) یا باقیمانده هاشون بر 3 1 و2 ( به پیمانه 3 بشن 1و 2) که این هم9 حالت داره ( 3تا برای باقیمانده 2 و 3 تا برای 1) پس در کل میشه: گزینه:4)12


سوال 3:
برای سطر اول 6 حالت داریم( 3 برای مهره سیاه و2 برای مهره سفید.) حالا توی سطر دوم مهره سیاه یا زیر سفیده یا زیر خونه خالی. در حالت اول خونه خالی باید زیر مهره سیاه باشه ( چون بنا بر لانه کبوتری اگه نباشه تو سطر آخر یکی از سیاه و سفید توی ستونی که قبلا از اونها داشته میان ( در واقع مهره سیاه و سفید فقط می تونن توی ستون دارای دوتا بدون مهره بیان) اگر هم توی سطر دوم مهره سیاه زیر بدون مهره بیاد برای سفید فقط یک جا می مونه. واضحه که برا سطر آخر هم فقط یه حالت داریم. پس جواب میشه6*2=12(6 سطر اول و 2 سطر دوم) یعنی :گزینه:2)12


سوال4:
واضحه جواب حداکثر 8 هست. من راهی می دم که جواب همین بشه( شرمنده عکس نذاشتم عدد های یکسان یک کاشی اند.): پس جواب میشه: گزینه:4)8
1 1 1 2 3
4 4 4 2 3
5 6 * 2 3
5 6 7 7 7
5 6 8 8 8
ممنون از راه حلتون...
شما دارین برای المپیاد می خونید؟
می تونم بپرسم که کلاس چندمید؟

yugioh
09-09-2007, 14:14
خواهش می کنم مرسی از لطفتون. راستش من قبلا واسه المپیاد می خوندم. البته الان نه دیگه. سنم یک کم از این کار گذشته ول خب ریاضی رو تاحدی دوست دارم. از شما هم ممنونم.

pp8khat
09-09-2007, 15:02
خواهش می کنم مرسی از لطفتون. راستش من قبلا واسه المپیاد می خوندم. البته الان نه دیگه. سنم یک کم از این کار گذشته ول خب ریاضی رو تاحدی دوست دارم. از شما هم ممنونم.

ببخشید اینقدر سوال می پرسما!!
با اجازه ی آقای مفیدی فقط یه سوال دیگه می پرسم(آخه من سابقم تو چرت و پرت نوشتن تو تاپیک ها و انحراف زیاده!)
شما وقتی که داشتید واسه ی المپیاد می خوندید از چه کتاب استفاده می کردید؟
ممنون

pp8khat
17-09-2007, 18:38
با سلام. چند تا سوال داشتم:-)
1-
میشه یکی یه کم درباره اندازه گیری سطح زیر نمودار توضیح بده؟؟
مثلاً سطح زیر سهمی رو چه جوری اندازه می گیرن؟؟
با عدد "پی" سر و کار داره؟؟
مرز اندازه گیری محور x'ox و y'oy هستش؟؟
میشه این مرز رو به یه خط دیگه انتقال داد؟؟
2-
این سوال رو از خیلی وقت پیش ها داشتم و سال اول برام پیش اومده بود که تو پاورقی کتاب ریاضی 1 خونده بودم...
نوشته بود " بعضی از کتاب ها عدد 0.25 که تحقیقی است را به صورت 0.25000.... یا 0.250(بالای صفر خط=یعنی
صفر در گردش) و یا 0.2499999..... یا 0.249(بالای نه خط=یعنی نه در گردش) می نویسند و می گویند که هر عدد
گویا دارای بسط اعشاری متناوب است."
ولی نویسنده نگفت که این کتاب ها کار درستی می کنند یا خیر؟؟
3-من کلاً " چند تا...می توان یافت" ها رو مشکل دارم...
چند عدد طبیعی را می توان به صورت جمع تعدادی عدد طبیعی متمایز کمتر از 100 نوشت؟
الف-4950 ب-1440 ج-4851 د-5050 ه-بیش از 10000
4-
یک غلتک مخروطی شکل رنگی،با زاویه راس 60 درجه و طول یال یک متر،یک دور روی سطح زمین غلت می خورد.با
فرض این که راس مخروط ثابت است،مساحت رنگ شده سطح زمین چند متربربع است؟(نیاز شدید به شکل دارم)
الف)1 ب)2/پی ج) 2/پی ضرب در رادیکال 3 د) پی ه)0.5
5-
می دانیم که نمایش !20 در مبنای 10 به این صورت است:24329020081766xy000 که x و y رقمند.کدام درست
است؟
الف)x=0,y=0
ب)x=5,y=8
ج)x=8,y=5
د)x=4,y=0
ه)x=8,y=0
6-
A,B و C سه زیر مجموعه دلخواه مجموعه اعداد طبیعی هستند.با دو عمل اجتماع و مکمل(مکمل دیگه چیه؟؟؟)،حداکثر چند مجموعه مختلف می توان ساخت؟
الف)7 ب)8 ج)18 د)128 ه)256
7-
مجموع عناصر جدول ضرب 10*10 چند است.در صورتی که در ستون i و سطر j این جدول i*j نوشته شده باشد؟

SuB
17-09-2007, 20:55
با سلام. چند تا سوال داشتم:-)
1-
میشه یکی یه کم درباره اندازه گیری سطح زیر نمودار توضیح بده؟؟
مثلاً سطح زیر سهمی رو چه جوری اندازه می گیرن؟؟
با عدد "پی" سر و کار داره؟؟
مرز اندازه گیری محور x'ox و y'oy هستش؟؟
میشه این مرز رو به یه خط دیگه انتقال داد؟؟

اول بگم که سطح زیر نمودار الزاماً زیر نمودار قرار نداره. بلکه سطح زیر نمودار تابع f در بازه [a,b] برابر است با مساحت محصور بین خط‌های x=a و x=b و y=0 (محور xها) و نمودار تابع f در آن بازه.

برای تابع‌های ساده مثلا تابع همانی (با ضابطه y=x)، به راحتی با استفاده از روابط هندسی، مساحت زیر نمودار رو اندازه گیری کنید.

مساحت زیر نمودار توابع با انتگرال معین توابع رابطه بسیار نزدیکی داره. در واقع سطح زیر نمودار برابر با قدر مطلق انتگرال اون تابع هست.

برای محاسبه انتگرال معین دو روش هست. 1- استفاده از تعریف انتگرال معین 2- استفاده از انتگرال نامعین و قضیه 1 و 2 اساسی حساب انتگرال. (انتگرال نامعین با انتگرال معین خیلی تفاوت داره و فکر نکنید انتگرال نامعین مساحت زیر نمودار تابع است که در بازه مشخصی داده نشده است! یا این جور چیزها (شنیدم که دارم میگم!!!!))
روش اول بسیار وقت گیر و گاهی غیر ممکن است. ولی روش دوم بسیار ساده و کوتاه هست. البته با روش دوم نمیشه انتگرال معین هر تابع رو مشخص کرد.

مرز اندازه گیری رو به راحتی می‌تونید تغییر بدید. حتی شما با استفاده از انتگرال معین می‌تونید مساحت محصور میان نمودار دو تابع رو بدست بیارید. یا حجم حاصل از دوران نمودار یک تابع رو توی 3 بعد محاسبه کنید.

برای درک انتگرال معین نیاز به یادگیری دست کم حد و مشتق و دنباله دارید. (2 تای اولی می‌تونند پدر آدم رو در بیارند)

شما بهتر بود این سوال رو توی اتاق حساب دیفرانسیل و انتگرال مطرح می‌کردید. چون این سوال مربوط به اون تاپیک میشه. اگه سوال دیگه‌ای در این زمینه هست لطفاً توی همون تاپیک مطرح کنید.

pp8khat
27-09-2007, 17:33
اول بگم که سطح زیر نمودار الزاماً زیر نمودار قرار نداره. بلکه سطح زیر نمودار تابع f در بازه [a,b] برابر است با مساحت محصور بین خط‌های x=a و x=b و y=0 (محور xها) و نمودار تابع f در آن بازه.

برای تابع‌های ساده مثلا تابع همانی (با ضابطه y=x)، به راحتی با استفاده از روابط هندسی، مساحت زیر نمودار رو اندازه گیری کنید.

مساحت زیر نمودار توابع با انتگرال معین توابع رابطه بسیار نزدیکی داره. در واقع سطح زیر نمودار برابر با قدر مطلق انتگرال اون تابع هست.

برای محاسبه انتگرال معین دو روش هست. 1- استفاده از تعریف انتگرال معین 2- استفاده از انتگرال نامعین و قضیه 1 و 2 اساسی حساب انتگرال. (انتگرال نامعین با انتگرال معین خیلی تفاوت داره و فکر نکنید انتگرال نامعین مساحت زیر نمودار تابع است که در بازه مشخصی داده نشده است! یا این جور چیزها (شنیدم که دارم میگم!!!!))
روش اول بسیار وقت گیر و گاهی غیر ممکن است. ولی روش دوم بسیار ساده و کوتاه هست. البته با روش دوم نمیشه انتگرال معین هر تابع رو مشخص کرد.

مرز اندازه گیری رو به راحتی می‌تونید تغییر بدید. حتی شما با استفاده از انتگرال معین می‌تونید مساحت محصور میان نمودار دو تابع رو بدست بیارید. یا حجم حاصل از دوران نمودار یک تابع رو توی 3 بعد محاسبه کنید.

برای درک انتگرال معین نیاز به یادگیری دست کم حد و مشتق و دنباله دارید. (2 تای اولی می‌تونند پدر آدم رو در بیارند)

شما بهتر بود این سوال رو توی اتاق حساب دیفرانسیل و انتگرال مطرح می‌کردید. چون این سوال مربوط به اون تاپیک میشه. اگه سوال دیگه‌ای در این زمینه هست لطفاً توی همون تاپیک مطرح کنید.
یه دنیا ممنون
راجع به قسمت قرمز:گفتم اسم این تاپیک تاپیک طرح سوالاته برای همین اینجا مطرحش کردم...
دوستان سوالا سخت بود بقیه رو جواب ندادین؟؟؟
ولی عوضش الان یه سوال المپیادیه 5 گزینه ایه استاندارد شده درست کردم؛می خوام ببینم کدومتون می تونید جواب درست رو بدید!!
سوال:
احتمال اینکه همین حالایی که شما در حال خواندن این سوال هستید،زلزله بیاید و سقف روی سرتان خراب شود و شما با سرعت نور به سمت جهت مثبت محور y ها حرکت کنید(بمیرید!!!)،چند درصد است؟
1)25%
2)33.3%
3)50%
4)75%
5)نمی توان تعیین نمود

منتظرم:5:

SuB
27-09-2007, 20:49
دوستان سوالا سخت بود بقیه رو جواب ندادین؟؟؟
این سوال‌ها که در تخصص من نبود. بعضی‌هاش رو بلدم جواب بدم. ولی حوصله و وقت حلش رو ندارم. بهتره بقیه دوستان صاحب‌نظر جواب بدند.


ولی عوضش الان یه سوال المپیادیه 5 گزینه ایه استاندارد شده درست کردم؛می خوام ببینم کدومتون می تونید جواب درست رو بدید!!
سوال:
احتمال اینکه همین حالایی که شما در حال خواندن این سوال هستید،زلزله بیاید و سقف روی سرتان خراب شود و شما با سرعت نور به سمت جهت مثبت محور y ها حرکت کنید(بمیرید!!!)،چند درصد است؟
1)25%
2)33.3%
3)50%
4)75%
5)نمی توان تعیین نمود
ایول به سوال.:2::27:

اول بگو سه بعدی در نظر گرفتی یا دو بعدی؟ (اصلاً می‌خوام بدونم کدوم محور رو محور y ها در نظر گرفتید؟):21:

شوخی بود:46:

pp8khat
27-09-2007, 21:12
چی؟
سوال؟
سوالم جدی بودااا!!
راه حلشم خیلی خفنه!!
عزیز همون دو بعدی خودمون که عمود بر محور x'ox هستش محور y دیگه!!!!
پسر برو بیشتر تست بزن و درس بخون(شوخی!!)

the dead
02-10-2007, 12:34
سلام سوالی داشتم
[ برای مشاهده لینک ، لطفا با نام کاربری خود وارد شوید یا ثبت نام کنید ]
در قسمت خطوط قرمز آيا اگر مقدار بازه راست در چپ ضرب شود و کوچکتر از صفر شود هميشه يک ريشه دارد
اگر قضايای ديگری نيز صادق است نيز بفرماييد

SuB
02-10-2007, 20:34
سلام سوالی داشتم
[ برای مشاهده لینک ، لطفا با نام کاربری خود وارد شوید یا ثبت نام کنید ]
در قسمت خطوط قرمز آيا اگر مقدار بازه راست در چپ ضرب شود و کوچکتر از صفر شود هميشه يک ريشه دارد
اگر قضايای ديگری نيز صادق است نيز بفرماييد

در این مورد خاص یک ریشه شده. ولی به طور کلی اگه حاصلضرب مقدار راست بازه در مقدار چپ بازه کوچکتر از صفر بشه، حداقل یه ریشه داره.

در این مورد چون تابع درجه 3 بود و مشتقش مثبت هست یعنی تابع اکیداً صعودی هست، پس این تابع تنها یه صفر داره ولی چون حاصلضرب (f(1 در (f(-1 منفی هست، پس این یه ریشه حتماً در همین بازه هست.

the dead
03-10-2007, 08:08
در این مورد خاص یک ریشه شده. ولی به طور کلی اگه حاصلضرب مقدار راست بازه در مقدار چپ بازه کوچکتر از صفر بشه، حداقل یه ریشه داره.

در این مورد چون تابع درجه 3 بود و مشتقش مثبت هست یعنی تابع اکیداً صعودی هست، پس این تابع تنها یه صفر داره ولی چون حاصلضرب (f(1 در (f(-1 منفی هست، پس این یه ریشه حتماً در همین بازه هست.

منظورتون چیه که اين تابع حتماً يک صفر داره

SuB
03-10-2007, 16:05
منظورتون چیه که اين تابع حتماً يک صفر داره

صفرهای یک تابع، x هایی از دامنه اون تابع هستند که به ازای آنها مقدار تابع صفر می‌شود. مثلاً تعداد صفرهای تابع‌های درجه 2 حداکثر برابر 2 و تعداد صفرهای تابع درجه 3 دست‌کم 1 و حداکثر 3 تاست.

بهتر بود این سوال رو توی اتاق حساب دیفرانسیل و انتگرال مطرح می‌کردید.:11:

sarooman
04-10-2007, 03:19
با عرض سلام به همه ی ریاضی دوستان عزیز

به خصوص آقای مفیدی

من هم ورودم را با ساده ترین سوال سخت شروع میکنم

ثابت کنید هرعدد حقیقی غیر صفر ، به توان صفر برابر یک می باشد

سحر علیخانی
04-10-2007, 21:00
سوال من در مورد بحث مجموعه ی اعداد است.چرا فقط یک مجموعه ی تهی وجود دارد؟و اینکه جرا تنها یک مجموعه ی مرجع برای تمام مجموعه ها وجود دارد؟با تشکر فراوان

SuB
05-10-2007, 12:14
سوال من در مورد بحث مجموعه ی اعداد است.چرا فقط یک مجموعه ی تهی وجود دارد؟و اینکه جرا تنها یک مجموعه ی مرجع برای تمام مجموعه ها وجود دارد؟با تشکر فراوان

اگه بخواد دو یا بیش از یک مجموعه مرجع وجود داشته باشه یا با هم مساوی هستند که در این صورت متمایز نیستند و در این صورت چند مجموعه نبوده و یه مجموعه بوده. اگه بخواند مساوی نباشند، عضوی توی یکی هست که تو دیگری نیست، پس یکی‌شون مرجع نیست چون تمام اعداد و مجموعه‌ها رو در بر نگرفته. پس مجموعه مرجه یکتاست.

محمد88
11-10-2007, 21:00
یه سوال ساده دارم اگه میشه با توضیح بگین :

ثابت کنید رادیکال 3 اصم است.(با توضیح)

Zokoo
11-10-2007, 21:57
یه سوال ساده دارم اگه میشه با توضیح بگین :

ثابت کنید رادیکال 3 اصم است.(با توضیح)
جوون ترکه بودیم فکرکنم بابرهان خلف اثبات میکردیم!!!:10:

e_kh78
14-10-2007, 14:44
لطفا راهنمایی کنید جواب این چند مساله
1.حد X+1 بر روی X-1 کل عبارت کسری به توان 2X-1 وقتی X به سمت مثبت بینهایت میل میکند چه جوری میشه e به توان 4
2.حد X+1 بر روی X-2 کل مجموعه کسری به توان 2X-1 وقتی X به سمت مثبت بینهایت میل میکند چه جوری میشه e به توان 6

لطفا اگر ممکن است واضح توضیح دهید چون جواب را دارم ولی متوجه نمیشوم

boogiman
15-10-2007, 12:45
سلام راستش امروز یک نفر از من این سوال رو پرسید:
دامنه و برد f(-1)chi میشه؟
منم موندم اگه میشه شما کمکم کنید
باتشکر:20:

پاکر
15-10-2007, 16:33
ویرایش شد...........

boogiman
15-10-2007, 18:32
خیلی ممنون از کمکتون ولی اون( 1- ) که گذاشتم به جای x هستش یعنی( f(x نه f وارون